You are on page 1of 113

कें द्रीय विद्यालय संगठन

(जम्मू संभाग )

KENDRIYA VIDYALAYA SANGATHAN


(JAMMU REGION)

SAMPLE PAPER
FOR CLASS X
MATHEMATICS
BASIC (241)
क्षेत्रीय कायाालय, जम्मू, नज़दीक राजकीय विककत्सालय, गााँधी नगर
जम्मू-180004
Regional Office, Jammu, Near Govt. Hospital,
Gandhi Nagar, Jammu-180004
KENDRIYA VIDYALYA SANGATHAN, JAMMU REGION
Sample Paper Set - 1

CLASS – X SUBJECT -Mathematics Basic (241)


Time : 3 Hours MM: 80

GENERAL INSTRUCTIONS:
Read the instructions carefully :
1. This Question Paper has 5 Sections A, B, C, D, and E.
2. Section A has 20 Multiple Choice Questions (MCQs) carrying 1 mark each.
3. Section B has 5 Short Answer-I (SA-I) type questions carrying 2 marks each.
4. Section C has 6 Short Answer-II (SA-II) type questions carrying 3 marks each.
5. Section D has 4 Long Answer (LA) type questions carrying 5 marks each.
6. Section E has 3 Case Based integrated units of assessment (4 marks each) with sub-parts of
the values of 1, 1 and 2 marks each respectively.
7. An internal choice in 2 Qs of 2 marks, 2 Qs of 3 marks and 2 Questions of 5 marks has
been provided.

SECTION A (1M EACH)


Q1. The sum of exponents of prime factors in the prime factorisation of 196 is
A) 3 B) 4 C) 5 D) 2

Q2. If two positive integers A and B can be expressed as A = xy3 and B = x4 y2z; x, y being
prime numbers then HCF (A, B) is

A) x𝑦 2 B) 𝑥 4 𝑦 2 z C) 𝑥 4 𝑦 3 D) 𝑥 4 𝑦 3 z

Q3. In the following question, a statement of assertion (A) is followed by a statement of


reason (R).
(A) HCF (23, 53) is 1.
(R) If p and q are prime then HCF (p, q) = 1

Now mark the correct choice as:


(a) Both assertion (A) and reason (R) are true and reason (R) is the correct explanation of
assertion (A).
(b) Both assertion (A) and reason (R) are true but reason (R) is not the correct explanation of
assertion (A).
(c) Assertion (A) is true but reasons (R) is false.
(d) Assertion (A) is false but reasons (R) is true.

Q4. If sum of the zeroes of the quadratic polynomialk𝑥 2 +2x+3k is equal to the product of its
zeroes , then the value of k is:

A) 1/3 B) -1/3 C) 2/3 D) -2/3

Q5. For what value of ‘k’ will the following pairs of linear equations are inconsistent

3x+4y=1 and (2k-1) x+(k-1) y = 2k+1

A) K=1 B) k=2 C) k=3 D) k=4

Q6. Which of the following equations has 2 as a root?

A) 𝑥 2 -4x+5=0 B) 𝑥 2 +3x-12=0 C) 2𝑥 2 -7x+6=0 D) 3𝑥 2 -6x-2=0

Q7. The ratio in which the X -axis divides the line segment joining A( 2,-3) and B(5,6) is :

A) 2:3 B) 3:5 C) 1:2 D) 2:1

Q8. Distance between the pairs of points: (a, b) and (-a,-b) is:

A) 2√𝑎2 + 𝑏2 B) √𝑎2 + 𝑏2 C) 0 D) √2𝑎2 + 2𝑏2

Q9. In the following question, a statement of assertion (A) is followed by a statement of


reason (R).
Assertion:- The points A (-2, 1), B (1, 0) and C (4, – 1) are collinear.
Reason:- Three points A, B and C are collinear in same straight line, if AB + BC = AC.

Now choose the correct option among following:


(A) Both assertion (A) and reason (R) are true and reason (R) is the correct explanation of
assertion (A).
(B) Both assertion (A) and reason (R) are true but reason (R) is not the correct explanation of
assertion (A).
(C) Assertion (A) is true but reasons (R) is false.
(D) Assertion (A) is false but reasons (R) is true.

Q10. If cos A= 4/5, then the value of tan A is

A) 3/5 B) 3/4 C) 4/3 D) 5/3

Q11. If ∆ABC is right angled at C , then the value ofcos (A +B) is:
A) 0 B) 1 C) 1/2 D)
Q12. If sinα=1/2 and cosβ=1/2, then the value of (α+β) is:
A) 00 B) 300 C) 600 D) 900

Q13. It is given that, ∆ ABC ~∆ 𝐸DF such that AB = 5 cm, AC=7cm, DF=15cm and

DE= 12 cm.then the length of EF is:

A) 17 cm B) 16.8 cm C) 6.25 cm D) 24 cm

Q14. Two chords AB and CD of a circle intersect at E such that AE = 2 .4cm, BE = 3. 2cm
and CE = 1. 6cm. The length of DE is:

A) 1.6 cm B) 3.2 cm C) 4.8 cm D) 6.4 cm

Q15. In a circle of radius 14 cm, an arc subtends an angle of 450 at the centre , then the area
of the sector is:

A) 71 𝑐𝑚2 B) 76 𝑐𝑚2 C) 77 𝑐𝑚2 D) 154 𝑐𝑚2

Q16. If the circumference of a circle increases from 4 π to 8 π , then its area is :

A) Halved B) doubled C) tripled D) quadrupled

Q17. The base radii of a cone and a cylinder are equal. If their curved surface areas are also
equal, then the ratio of the slant height of the cone to the height of the cylinder is:

A) 2:1 B) 1:2 C) 1: 3 D) 3:1

Q18. The median and mode respectively of a frequency distribution are 26 and 29, Then its
mean is:

A) 27.5 B) 24.5 C) 28.4 D) 25.8

Q19. For finding the popular size of readymade garments, which central tendency is used?

A) Mean B) Median C) Mode D) Both mean and mode

Q20. Which of the following cannot be the probability of an event?

A) 1/3 B) 0.1 C) 3% D) 17/16

SECTION B(2M EACH)

Q21. A fraction becomes 1/3 when 2 is subtracted from the numerator and it becomes ½
when 1 is subtracted from the denominator. Find the fraction.

Q22. Evaluate 𝑠𝑖𝑛2 602 -2tan450 -𝑐𝑜𝑠 2 300 .

OR

If Sin(A+B) =√3/2 And tan(A-B)=1/√3; 00 <A+B≤900 ; A>B, Find A and B.


Q23. In ∆ABC, DE || BC, find the value of x.

Q24. If PT and QT are tangents to the circle. Find ∠PTQ.

Q25. The radii of two circles are 8 cm and 6 cm respectively. Find the radius of the circle
having its area equal to the sum of the areas of the two circles.
OR
Find the angle subtended at the centre of a circle of radius 5 cm by an arc of length 5π/3
cm.
SECTION C(3M EACH)

Q26. Prove that √3is irrational.

Q27. Find the zeroes of the quadratic polynomial 6𝑥 2 -3-7x and verify the relationship
between the zeroes and the coefficients.

OR

If α and β are the zeroes of the polynomial 6𝑥 2 -7x+2, find a quadratic polynomial whose
zeroes are 1/α and 1/β

Q28. The sum of the squares of two consecutive odd numbers is 394. Find the numbers.

OR

Find the values of k for which the quadratic equations (k+4) 𝑥 2 +(k+1)x+1=0 has equal
roots.

Q29. Prove that (sinA +cosecA)2+(cosA+secA)2=7+𝑡𝑎𝑛2 𝐴+𝑐𝑜𝑡 2 A


Q30. In the given figure, PQ is a chord of a circle with centre O and PT is a tangent. If ∠QPT
= 60°, find ∠PRQ.

Q31. One card is drawn from a pack of 52 cards. What is the probability of getting?
(i) an ace ?
(ii) a red card ?
(iii) a face card?
SECTION D(5M EACH)

Q32. The diagonal of a rectangular field is 60 m more than the shorter side . If the longer side
is 30 m more than the shorter side , then find the sides of the field.
OR
The difference of squares of two numbers is 180. The square of the smaller number is
8 times the larger number . Find the two numbers.

Q33. State and prove Basic Proportionality theorem.

Q34. A Gulabjamun , contains sugar syrup upto 30% of its volume. Find approximately
how much syrup would be found in 45 gulabjamuns, each shaped like a cylinder with two
hemispherical ends with length 5 cm and diameter 2.8 cm

OR
From a solid cylinder whose height is 2.4 cm and diameter is 1.4 cm , a conical cavity of the
same height and same diameter is hollowed out . find the total surface area of the remaining

solid to the nearest cm sq.

Q35. The median of the following data is 525. Find the values of x and y if the total
frequency is 100.

Class Frequency
Interval
0-100 2
100-200 5
200-300 X
300-400 12
400-500 17
500-600 20
600-700 Y
700-800 9
800-900 7
900-1000 4

SECTION E(4M EACH)

CASE BASED

Q36. Seating Capacity : The Fox Theater creates a “theater in the round” when it shows any
of Shakespeare’s plays. The first row has 80 seats, the second row has 88, the third row has
96, and so on.
(i) How many seats are in the 10th row? (1)

(ii) How many seats are in the 25th row? (1)

(iii) If there is room for 25 rows, how many chairs will be needed to set up the theatre? (2)

OR

If nth row has 240 seats, find value of n.

Q37. Ajay, Bhigu and Colin are fast friend since childhood. They always want to sit in a row
in the classroom . But teacher doesn’t allow them and rotate the seats row-wise everyday.
Bhigu is very good in maths and he does distance calculation everyday. He consider the
centre of class as origin and marks their position on a paper in a co-ordinate system. One day
Bhigu make the following diagram of their seating position.
Now answer the questions: faces of the canyon is approximately 150 meter, they attempt

(i) What are the coordinates of point A? (1m)


(ii) What is the distance of point C from X-axis? (1 mark)
(iii) What is the distance between A and B ? (2m)

OR

A point D lies on the line segment between points A and B such that

AD :DB = 4:3.What are the coordinates of point D?(2M)


Q38. Height of a Climber : Himalayan Trekking Club has just hiked to the south rim of a
large canyon, when they spot a climber attempting to scale the taller northern face. Knowing
the distance between the sheer walls of the northern and southern faces of the canyon is
approximately 150 meter, they attempt to compute the distance remaining for the climbers to
reach the top of the northern rim. Using a homemade transit, they sight an angle of depression

of 60 deg. to the bottom of the north face, and angles of elevation of 30 deg. and 45 deg. to
the climbers and top of the northern rim respectively.

Now answer the following:

(i) How high is the southern rim of the canyon? (1)

(ii) How high is the northern rim? (1)


(iii) How much farther the climber has to go until the climber reaches top? (2)

OR

Find the distance between Observer and the climber.


1

KENDRIYA VIDYALYA SANGATHAN, JAMMU REGION


Sample Paper Set – 2

CLASS – X SUBJECT -Mathematics Basic (241)


Time : 3 Hours MM: 80
General Instructions:
1. This Question Paper has 5 Sections A-E.
2. Section A has 20 MCQs carrying 1 mark each
3. Section B has 5 questions carrying 02 marks each.
4. Section C has 6 questions carrying 03 marks each.
5. Section D has 4 questions carrying 05 marks each.
6. Section E has 3 case based integrated units of assessment (04 marks each) .
7. All Questions are compulsory. However, an internal choice in 2 Qs of 5 marks,
2 Qs of 3 marks and 2 Questions of 2 marks has been provided. An internal choice has
been provided in the 2marks questions of Section E
8. Draw neat figures wherever required. Take π =22/7 wherever required if not
stated.
SECTION- A
Q.No. MA
RKS
1 n2 -1 is divisible by8 if n is
(a) An integer (b) a natural number

(c)an odd number (d) an even number


1
2 If Sin2A = 2SinA, then A= ... º:
(a) 0 (b) 30 (c) 45 (d) 60

1
3 The product of two irrational number is

(a) Always irrational (b) Always rational

(c) rational or irrational (d) one


1
2
4 The sum and product of zeroes of polynomial 9x -5 respectively are 1
−9 9 5 −5
(a) 0, (b) 0, (c) 0, (d)0,
5 5 9 9

5 Pair of linear equation x =0 and y =-6 has 1

(a) Unique solution (b) no solution

(c) infinitely many solution (d) only solution (0,0)

6 If the equation x2 –mx +1 = 0does not possess real roots, then 1


2

(a) m>2(b) m< -2 (c) -2<m<2 (d) -3<m<3

7 The distance of the point P(3, -4) from the origin is 1


(a) 7 units (b) 5 units (c) 4 units (d) 3 units

8 The probability that a number selected at random from the numbers 1, 1


2, 3, ......, 15 is a multiple of 3 is
𝟐 𝟏 𝟏 𝟏
(a) (b) (c) (d)
𝟏𝟓 𝟓 𝟏𝟓 𝟑

9 If M is the mid-point of PQ, if P is ( 4 ,x) M is ( y ,- 1) and Q 1


is ( - 2 ,4) then x and y respectively are
(a) -6 and 1 (b) -6 and 2
(c) 6 and -1 (d) 6 and -2

10 𝟏 𝟏 1
If sin α = 𝟐and cosβ = 𝟐 then the value of α+ β is
(a) 0° (b) 30° (c) 60° (d) 90°

11 If x = rsinA and y = rcosA then x2 + y2 is 1


(a) r (b) r2 (c) 1/r (d) 1

12 The class mark of the classes 120 – 150 is 1


(a) 120 (b)130 (c) 135 (d) 150

13 In fig. if TP and TQ are two tangents to a circle with centre O then ∠𝑃𝑇𝑄 is 1
equal to

(a)500 (b) 600 (c) 700 (d) 800

14 Which of the following are not always similar: 1


(A)two circles (B) two squares (C) two equilateral triangles
(D) two rectangles

15 If the area of a circle is 154cm2 then perimeter is 1


3

(a) 11cm (b) 22cm (c) 44cm (d) 55cm

16 Which is equivalent to π 1
(a) Circumference /radius (b) Circumference /diameter

(c) Circumference X diameter (d) Circumference X radius

17 The radius of a sphere (in cm) whose volume is 12π cm2, is 1


(a) 3 cm (b) 3√3cm (c) 32/3 cm (d) 31/3 cm

18 The mean of the following frequency distributions 5. Then the value of p 1


is:-
xi 2 4 6 10 p
fi 3 2 2 1 1

(a) 4 (b) 6 (c) 7 (d) none of these

19 Assertion:The distance point P(2,3) from the x-axis is 3. 1

Reason: The distance from x-axis is equal to its ordinate


(a) Both assertion (A)and reason (R) are true and reason (R) is the
correct explanation of assertion (A)
(b) Both assertion (A)and reason (R) are true and reason (R) is not the
correct explanation of assertion (A)
(c) Assertion (A) is true but reason (R) is false.
(d) Assertion (A) is false but reason (R) is true.

20 The HCF of two numbers is 5 and theirproduct is 150, then their LCM is 30 1
Reason : For any two positive integers a and b,
HCF(a,b) = (LCM (a,b)= a X b
(a) Both assertion (A)and reason (R) are true and reason (R) is the
correct explanation of assertion (A)
(b) Both assertion (A)and reason (R) are true and reason (R) is not the
correct explanation of assertion (A)
(c) Assertion (A) is true but reason (R) is false.
(d) Assertion (A) is false but reason (R) is true.

SECTION- B

21 Solve 2x + y = 5 and 3x + 2y = 8 by method of Elimination by equating the 2


4

coefficient

22 Evaluate sin 60° cos 30° + sin 30° cos 60° 2


OR
If 5sin A = 3. Find values of cos A and tan A.

23 ABFE is a trapezium with AB I I EF. D and C are points on non parallel 2


sides AE and BF respectively such that DC is parallel to AB .Show
𝑨𝑫 𝑩𝑪
that =
𝑬𝑫 𝑭𝑪

24 In figure below, ΔABC is circumscribing a circle. Find the length of BC 2

OR
If (1, 2), (4, y), (x, 6) and (3, 5) are the vertices of a parallelogram taken in
order, find x and y

25 Find the area of the shaded region in the given figure, if ABCD is a square 2
of side 14 cm and APD and BPC are semicircles.

SECTION- C
26 Prove that √5 is an irrational number. 3
5

27 Find the zeros of quadratic polynomial 4x2 – 4x + 1 and verify the 3


relationship between Zeroes and its coefficients.

28 A train travels at a certain average speed for a distance of 63 km and then 3


travels adistance of 72 km at an average speed of 6 km/h more than its
original speed. If ittakes 3 hours to complete the total journey, what is its
original average speed?
OR
Find the roots of the equation 3x2-7x +2 = 0 by using quadratic formula

29 If tan  + sin  = m and tan  - sin  = n. Show that m2 - n2= . 3


OR
Evaluate
5 cos2 60o + 4 Sec2 30o - tan 45o
Sin2 30o + Cos2 30o

30 Find the diameter of the circle whose area is equal to the sum of the areas 3
of the circles of diameter 20cm and 48cm.

31 The king, queen and jack of clubs are removed from a deck of 52 playing 3
cards and remaining cards are shuffled. A card is drawn from the
remaining cards. Find the probability of getting a card of (i) heart (ii)
queen (iii) clubs.

SECTION- D
32 A train travels at a certain average speed for a distance of 63 km and then 5
travels adistance of 72 km at an average speed of 6 km/h more than
its original speed. If ittakes 3 hours to complete the total journey,
what is its original average speed?
OR

A motor boat whose speed is 18 km/h in still water takes 1 hour more to go
24 km upstream than to return downstream to the same spot. Find the
speed of the stream
33 If a line is drawn parallel to one side of a triangle to intersect the other two 5
sides in distinct points prove that the other two sides are divided in the
same ratio
34 From a solid cylinder whose height is 8 cm and of base radius 6 5
cm, a conical cavity of same height and same radius ishollowed
out. Find the volume and the total surface area of the Remaining
solid.(Take 𝜋= 3.1416).
OR

A solid is in the form of a cylinder with hemispherical ends. The total


height of the solid is 58 cm and the diameter of the cylinder is 28 cm. Find
the total volume of the solid. (Use π =22/7)

35 5
The mode of the following frequency distribution is 55.
6

Find the value of f1 and f2.

Class 0 - 10 15 - 30 30 - 45 45 - 60 60 - 75 75 - 90 Total
Interval

Frequency 6 7 f1 15 10 f2 51

SECTION- E
36 Mr. Rajesh Sharma aged 50 years is a senior Mathstescher living in New 4
Delhi. As per the Government directions he had taken his first dose of
COVID-19 vaccines in the months of April 2021in a city hospital.
Now he wants to take his second dose of the vaccine.
The following map is showing three vaccination centers around his home
. Let O represents his home. Vaccination centers are at A, B, and C as
shown in the figure below

Based on the above information answer the following questions


Q1.What are the coordinates of nearest vaccination center? (1 mark)

Q2.What is the distance of vaccination center C from his home? (1 mark)


Q3.In what ratio the vaccination center B is dividing the line joining
vaccination center A to C ? (2 marks)
OR

Find the distance between the vaccination centers A and C ? (2 marks)

37 In a school the students wanted to plant trees in and around the school to 4
reduce air pollution. It was decided that the number of trees that each
section of each class will plant will be equal to the class in which they are
studying.If there are 1 to 12 classes in the school and each class has 4
sections.
Based on the above information answer the following questions
Q1. Write the arithmetic progression formed for number of trees planted
7

for the above case. (1 mark)


Q.2. Find the common difference for above Arithmetic progression. (1
mark)

Q3.Find the number of trees planted by each class12. (2 Marks)


OR
Q4. What is the total number of trees planted?(2 marks)
38 Repairing of Electric Fault 4
An electrician has to repair an electric fault on the pole (AB) of
height of 8m.He needs to reach a point 2m (AC) below the top
of the pole to undertake the repair. Ladder makes an angle of
600with the ground.

Based on the above information answer the following questions


1. Find the Length of BC. (1 mark)
2. Find the measure of angle BCD. (1 mark)
3. Find the length of BD. (2 marks)
OR
Find the length of Ladder. (2 marks)
KENDRIYA VIDYALYA SANGATHAN, JAMMU REGION

Sample Paper Set – 3

CLASS – X SUBJECT -Mathematics Basic (241)

Time : 3 Hours MM: 80


Read the instructions carefully:
1. This Question Paper has 5 Sections A, B, C, D, and E.
2. Section A has 20 Multiple Choice Questions (MCQs) carrying 1 mark each.
3. Section B has 5 Short Answer-I (SA-I) type questions carrying 2 marks each.
4. Section C has 6 Short Answer-II (SA-II) type questions carrying 3 marks each.
5. Section D has 4 Long Answer (LA) type questions carrying 5 marks each.
6. Section E has 3 Case Based integrated units of assessment (4 marks each) with sub-parts of the values
of 1, 1 and 2 marks each respectively.
7. All Questions are compulsory. However, an internal choice in 2 Qs of 2 marks, 2 Qs of 3 marks and
2 Questions of 5 marks has been provided.

SECTION – A
Q1. The class interval of a given observation is 10 to 15, then the class mark for this
interval will be:
(a) 11.5
(b) 12.5
(c) 12
(d) 14

Q2. If the sum of given frequencies is 24, then the value of x in the observation: x,
5,6,1,2, will be;
(a) 4
(b) 6
(c) 8
(d) 10

Q3. For any two positive integers a and b, HCF (a, b) × LCM (a, b) =
(a) 1
(b) (a × b)/2
(c) a/b
(d) a × b

Q4. The probability of an event is always


(a) 0≤ P(E) ≤ 1
(b) 0≤ P(E) ≥1
(c) 0< P(E) < 1
(d) none of these

Q5. HCF of 26 and 91 is:


(a) 15
(b) 13
(c) 19
(d) 11

Q6. A cylindrical pencil sharpened at one edge is the combination of


(a) a cone and a cylinder
(b) two cylinders
(c) a hemisphere and a cylinder
(d) none of these

Q7. Which of the following is not irrational?


(a) (3 + √7)
(b) (3 – √7)
(c) (3 + √7) (3 – √7)
(d) 3√7

Q8. The area of a sector of a circle with radius 6 cm if the angle of the sector is 60°.
(a) 142/7
(b) 152/7
(c) 132/7
(d) 122/7

Q9. The length of a tangent from a point A at a distance 5 cm from the centre of the circle
is 4 cm. The radius of the circle is:
(a) 3 cm
(b) 5 cm
(c) 7 cm
(d) 10 cm

Q10. If θ is the angle (in degrees) of a sector of a circle of radius r, then the length of arc is
(a) (πr2θ)/360
(b) (πr2θ)/180
(c) (2πrθ)/360
(d) (2πrθ)/180

Q11. The degree of the polynomial, x4 – x2 +2 is


(a) 2
(b) 4
(c) 1
(d) 0

Q12. The midpoint of a line segment joining two points A(2, 4) and B(-2, -4) is
(a) (-2, 4)
(b) (2, -4)
(c) (0, 0)
(d) (-2, -4)

Q13. The pairs of equations 9x + 3y + 12 = 0 and 18x + 6y + 26 = 0 have


(a) Unique solution
(b) Exactly two solutions
(c) Infinitely many solutions
(d) No solution

Q14. If triangle ABC is similar to triangle DEF, then,


(a) AB/FD = BC/EF = CA/DE
(b) AB/DE = BC/DF = CA/EF
(c) AB/DE = BC/EF = CA/FD
(d) AB/BC = CA/DE = EF/FD

Q15. sec2A – tan2A is equal to:


(a) 0
(b) -1
(c) 1
(d) none of these

Q16. The quadratic equation 2x2 – √5x + 1 = 0 has


(a) two distinct real roots
(b) two equal real roots
(c) no real roots
(d) more than 2 real roots

Q17. The distance of point A(2, 4) from the x-axis is


(a) 2 units
(b) 4 units
(c) -2 units
(d) -4 units

Q18. sin 2A = 2 sin A is true when A =


(a) 30°
(b) 45°
(c) 0°
(d) 60°

Assertion & Reasoning (Q19 and 20)


DIRECTION : In the following questions, a statement of assertion (A) is followed by a
statement of reason (R). Mark the correct choice as:
(a)Both assertion (A) and reason (R) are true and reason (R) is the correct explanation
of assertion (A). (b)Both assertion (A) and reason (R) are true but reason (R) is not the
correct explanation of assertion (A). (c)Assertion (A) is true but reason (R) is false.
(d)Assertion (A) is false but reason (R) is true
Q19. Assertion: The point which divides the line joining the points A(1, 2) and B(- 1, 1)
internally
in the ratio 1: 2 is (−1 2 , 5 3 )
Reason: The coordinates of the point P(x, y) which divides the line segment joining
the points
𝑚𝑥2 +𝑛𝑥1 𝑚𝑦2 +𝑛𝑦1
A(𝑥1 , 𝑦1 ) and B(𝑥2 , 𝑦2) in the ratio 𝑚 : n is ( , )
𝑚+𝑛 𝑚+𝑛

Q20. Assertion: The H.C.F. of two numbers is 16 and their product is 3072. Then their
L.C.M. = 162.
Reason: If a and b are two positive integers, then H.C.F. × L.C.M. = a × b.

SECTION – B
Q21. If the lines 3x+2ky – 2 = 0 and 2x+5y+1 = 0 are parallel, then what is the value of k?

OR
Find the values of x and y in the following rectangle:

Q22. Find the value of sin 60° cos 30° + sin 30° cos 60°
Q23. Prove that the tangents drawn to a circle from an external point are equal.

Q24. In the figure, if PQ || RS, prove that


∆ POQ ~ ∆ SOR..

Q25. A cow is tied with a rope of length 14 m at the corner of a rectangular field of
dimensions 20m x 16 m. Find the area of the field in which the cow can graze.
OR
A chord subtends an angle of 90°at the centre of a circle whose radius is 20 cm.
Compute the area of the corresponding major segment of the circle.
SECTION – C
Q26. Prove that 8 + 2√3 is irrational.
Q27. Solve the quadratic equation 2x2 – 7x + 3 = 0 by using quadratic formula.
Q28. If a card is drawn from a well shuffled deck of 52 cards,Find the probability of getting:
(a)an Ace of red colour
(b)a 10 of club
(c)a face card of spade.
OR
A bag contains a red ball, a blue ball and a yellow ball, all the balls being of the same
size. Kritika takes out a ball from the bag without looking into it. What is the
probability that she takes out the
(i) yellow ball?
(ii) red ball?
(iii) blue ball?

Q29. Find the zeroes of the following polynomials by factorisation method and verify the
relations between the zeroes and the coefficients of the polynomials:
3x2–x–4
OR
Find the quadratic polynomial if its zeroes are 0, √5.
Q30 If sin θ + cos θ = √3, then prove that tan θ + cot θ = 1.
Q31. AB is a chord of the circle and AOC is its diameter such that angle ACB = 50°. If AT
is the tangent to the circle at the point A, then find ∠ BAT is equal to
SECTION – D
Q32. State and prove Thales theorem.
Q33. The mean of the following frequency distribution is 50 but the frequencies f1 and f2 in
classes 20-40 and 60-80, respectively are not known. Find these frequencies, if the
sum of all the frequencies is 120.
CI 0-20 20-40 40-60 60-80 80-100
F 17 f1 32 f2 19

OR
The median of the following data is 50. Find the values of p and q, if the sum of all the
frequencies is 90.
CI F
20-30 P
30-40 15
40-50 25
50-60 20
60-70 Q
70-80 8
80-90 10
Q34. In a flight of 600 km, an aircraft was slowed due to bad weather. Its average speed for
the trip was reduced by 200 km/hr and the time of flight increased by 30 minutes. Find
the original duration of the flight.
Q35. An ice-cream cone full of ice-cream having radius 5 cm and height 10 cm as shown in
figure .Calculate the volume of ice-cream, provided that its 1/6 part is left unfilled
with ice-cream

OR
A rocket is in the form of a right circular cylinder closed at the lower end and
surmounted by a cone with the same radius as that of the cylinder. The diameter and
height of the cylinder are 6 cm and 12 cm, respectively. If the slant height of the
conical portion is 5 cm, then find the total surface area of the rocket, (use π = 3.14)

SECTION – E

Q36. Your elder brother wants to buy a car and plans to take loan from a bank for his car.
He repays his total loan of Rs 1,18,000 by paying every month starting with the first
instalment of Rs 1000. If he increases the instalment by Rs 100 every month , answer
the following:
(a) Write the first four terms for the Arithmetic progression for the above situation.

(b) Find the amount paid by him in 30th installment ? (1 m)

(c) If total installments are 40 then find the amount paid in the last
installment?(2m)

OR

Find the amount paid by him in 30 installments ?(2m)

Q37. A School trip was organized to a bird sanctuary,a boy observed the following situation.

A bird is sitting on the top of a tree, which is 80 m high. The angle of elevation of the bird,
from a point on the ground is 45o. The bird flies away from the point of observation
horizontally and remains at a constant height. After 5 seconds, the angle of elevation of the
bird from the point of observation becomes 30o , (Take √3=1.73 ).
(a) Find the value of y
(b) Find the value of x+y .
(c) Find the value of x. (2m)
OR
Find the speed of the bird,when it flies from point C to D

Q38. Football is the most popular game in the world. It combines strength, speed, and skills.
Football is the most-watched and most played sport on the earth. In Canada and
America, it is called Soccer. Football was invented in China nearly 476 BC. It is a 90
minutes long game and those 90 minutes are distributed in two 45 minutes halves.
Three friends Rohit, Rahul, and Ramesh decide to play Football in a playground of the
School. The position of Rohit, Rahul, and Ramesh is shown in the diagram.

(a) What is the Co-ordinate of Ramesh's position on the field? (1m)


(b) What is the Co-ordinate of Rahul's position on the field? (1m)
(c)If Rahul takes Football to position E and Rohit comes at position G then find
the distance between them . (2m)
OR
Find the co-ordinates of mid point of DE. (2m)
KENDRIYA VIDYALYA SANGATHAN, JAMMU REGION

Sample Paper Set – 4

CLASS – X SUBJECT -Mathematics Basic (241)

Time : 3 Hours MM: 80


General Instructions:`
1. This Question Paper has 5 Sections A, B, C, D, and E.
2. Section A has 20 Multiple Choice Questions (MCQs) carrying 1 mark each.
3. Section B has 5 Short Answer-I (SA-I) type questions carrying 2 marks each.
4. Section C has 6 Short Answer-II (SA-II) type questions carrying 3 marks each.
5. Section D has 4 Long Answer (LA) type questions carrying 5 marks each.
6. Section E has 3 Case Based integrated units of assessment (4 marks each) with sub-parts of
the values of 1, 1 and 2 marks each respectively.
7. All Questions are compulsory. However, an internal choice in 2 Qs of 2 marks, 2 Qs of 3
marks
and 2 Questions of 5 marks has been provided. An internal choice has been provided in the 2
marks questions of Section E.
8. Draw neat figures wherever required. Take π =22/7 wherever required if not stated.
Section A

1. If HCF (26,169)=13,then LCM (26,169) is 1


(a) 26 (b) 52 (c) 338 (d) 13
2 The pair of linear equations x +2y +5 =0 and –3 x –6y +1 =0 have 1
(a) Unique solution (b) exactly two solutions
( c) infinitely many solutions (d) cannot be determined
3 Which of the following is not a quadratic equation? 1
(a) 2(x—1)2= 4x2 –2x +1 (b) 2x—x2 = x2 +5
( c) √2𝑥 + √3)2 +x2 = 3x2 – 5 x (d) (x2 +2x)2 = 4x4 + 3+4x3
4 If the discriminant of a quadratic equation is less than zero, then it has 1
(a) Equal roots (b) real roots
(b) No real roots (d) none of the above
5 The distance between the points A(0,6) and B(0,--2) is 1
(a) 6 units (b) 8 units (c) 4 units (d) 2 units

6. The x-axis divides the joining of (2,--3) and (5,6) in the ratio 1
(a) 1:2 (b) 2 : 1 (c) 2 : 5 (d) 5 : 2
7. If Sin A = ½, then the value of Cot A is 1
1 √3
(a) √3 (𝑏 ) (c) (d) 1
√3 2
8. 3−𝑠𝑖𝑛 2 60° 1
What is the value of is
tan 30°.𝑡𝑎𝑛60°
9 13 11 15
(a) (b) (c) (d)
4 4 4 4

9. If a and b are coprime numbers the H.C.F of a and b is 1


a) 0 b) 1 c) 2 d) none of the above

10. Raju is 6 feet tall. At an instant his shadow is 5 feet long . At the same instant the shadow of a pole 1
is 30 feet long. How tall is the pole?
(a) 12 feet (b) 24 feet (c) 30feet (d) 36 feet
11. In the given fig, TP and TQ are the tangents drawn with the centre at O, 1

If ∠ 𝑃𝑂𝑄 = 1150 then ∠𝑃𝑇𝑄 𝑖𝑠


(a) 1150 (b) 57.50 (c) 550 (d) 650
12. In the given figure, the perimeter of ∆𝐴𝐵𝐶 1

(a) 30 cm (b) 60 cm (c) 45 cm (d) 15cm


The diameter of a wheel is 1m. The number of revolution will make to travel a distance of 22km will 1
13 be.
(a) 2800 (b) 4000 (c) 5500 (d) 7000
2
14. If the area of a circle is 154 cm , then its perimeter is 1
(a) 11 cm (b) 22 cm (c) 44 cm (d) 55 cm

15. The radius of a sphere (in cm) whose volume is 12𝜋 cm3, is
3
(a) 3 cm (b) √3 𝑐𝑚 (c) 32/3 cm (d) 31/3 cm 1
16. If the mean of data is 27 and mode is 45 ,the median is 1
(a) 30 (b) 27 (c) 32 (d) 33
/17 For the following distribution 1
CI 0-5 5-10 10-15 15-20 20-25
Frequency 10 15 12 20 9
The sum of lower limits of the median class and model class is
(a) 15 (b) 25 (c) 30 (d) 35
18 A card is selected from a deek of 52 cards . The probability of being a red face card is 1
3 3 2 1
(a) 26 (b) 13 (c) 13 (d)2
Direction: In the following questions, a statement of Assertion (A) is followed by a statement of 1
Reason (R).
Mark the correct choice as:
(a) Both Assertion (A) and Reason (R) are true and Reason (R) is the correct explanation of Assertion(A).
(b) Both Assertion (A) and Reason (R) are true but Reason (R) is not the correct explanation of
Assertion (A).
(c)Assertion(A) is true but Reason (R) is false.
(d) Assertion (A) is false but Reason (R) is true.

19 Assertion: The H.C.F. of two numbers is 16 and their product is 3072. Then their L.C.M. = 192. 1
Reason: If a and b are two positive integers, then H.C.F. × L.C.M. = a × b.

20 .Assertion: cot A is the product of cot and A. 1


Reason: The value of sin A increases as A increases .

SECTION - B

21 Find a quadratic polynomial each with the given numbers as the sum and product of its zeroes 2
are 1/4 , -1
OR
If the sum of the zeros of the quadratic polynomial k𝑦 2 + 2y – 3k is equal to twice their product , find
k.

22. The mode of the following data is 36. Find x 2


Class 0-10 10-20 20-30 30-40 40-50 50-60 60-70
Frequency 8 10 X 16 12 6 7

23. BE BF 2
In the figure, DE||AC and DF||AE. Prove that = FE
EC

24. Prove that the parallelogram circumscribing a circle is a rhombus 2

25 The radii of two circles are 8 cm and 6 cm, respectively. Find the radius of the circle having area
equals to the sum of the areas of the two circles.
0R
The length of the minute hand of a clock is 14 cm. Find the area swept by the minute hand in 5
minutes.
SECTION : C

26. Prove that √2 is an irrational number 3

27. Write the coordinates of a point on the x-axis which is equidistant from points A(-2, 0) and B(6, 0). 3

28. If the roots of the quad equation (a-b)x2+(b-c)x+(c-a) = 0 are equal, prove that 3
2a=b+c
Or
Find the value of k, for which the quadratic equation (k- 12) x2 + 2 (k-12) x + 2 = 0 has equal roots .

29. Prove that sec A (1 – sin A)(sec A + tan A) = 1. 3


30 If a circle touches the sides BC of a triangle ABC at P and extended sides AB and AC at Q and R 3
1
respectively ,Prove that AQ= 2(BC+CA+AB)

31 A bag contains 5 red,8 white and 7 black balls. A ball is drawn at random from the bag. Find the 3
probability that the drawn ball is
(i) red or white (ii) not black (iii) neither red nor black

OR
From a pack of 52 playing cards ,Jacks, Queens, Kings and aces of red colour are removed. From the
remaining, a card is drawn at random. Find the probability that the card drawn is
(i) a black queen (ii) a red card (iii) a black jack

SECTION-D

32 A motor boat whose speed is 24 km/h in still water takes 1 hour more to go 32 km upstream than 5
to return downstream to the same spot. Find the speed of the stream.
OR
A train travels 180 km at a uniform speed. If the speed had been 9 km/ hour more, it would have
taken 1 hour less for the same journey. Find the speed of the train

33 From a solid cylinder whose height is 12 cm and diameter is 10 cm , a conical cavity of same height and 5
same diameter is hollowed out. Find the volume and total surface area of the remaining solid.

34 The median of the following distribution is 32.5 . Find the values of f1 and f2
Class 0-10 10-20 20-30 30-40 40-50 50-60 60-70 Total 5
Frequency f1 5 9 12 f2 3 2 40

35 Prove that, if a line is drawn parallel to our side of a triangle to intersect the other two sides in distinct 5
points. The other two sides are divided in the same ratio
OR
In the given figure, P is the mid point of BC and Q is the midpoint of AP. If BQ when produced meet AC at R ,
1
Prove that RA = 3 CA
SECTION -E

Your friend Veer wants to participate in a 200m race. He can current ly run that distance in 51 seconds and
36 with each day of practice it takes him 2 seconds less. He wants to do in 31 seconds .

1 Frame the AP for the given situation. 1


2. Is 30 the term of an A.P term is not in the AP of the above given situation
3. What is the minimum number of days he needs to practice till his goal is 1
achieved ?
2
OR
th
If n term of an AP is given by an = 2n + 3 then find the common difference of an AP .

37 Read the following passage and answer the questions that follows: In a class room, four students
Sita, Gita, Rita and Anita are sitting at A(3,4), B(6,7), C(9,4), D(6,1) respectively. Then a new
student Anjali joins the class
1. Teacher tells Anjali to sit in the middle of the four students. Find the coordinates of the position 1
where she can sit.
2. Calculate the distance between Sita and Anita. 1
3. Which two students are equidistant from Gita.
OR 2
Which two students are equidistant from Anita

38 Consider a telecom tower BC fixed on top of a building AB. The distance between the
base of the building and point P on the ground is 48 m. From the point P the angle of
elevation of the top of a building B is 300 and the angle of elevation of the top of the
tower C is 600 .

1. The angle of depression from the top of the tower to the point P is 1
OR

2.The angle of depression from the top of the building to the point P is
2
3. What is the height of the building AB ?

OR
Find the height of the telecom tower BC ?
KENDRIYA VIDYALYA SANGATHAN, JAMMU REGION
Sample Paper Set – 5

CLASS – X SUBJECT -
Mathematics Basic (241)
Time : 3 Hours
MM: 80
General Instructions:
1. This Question Paper has 5 Sections A, B, C, D, and E.
2. Section A has 20 Multiple Choice Questions (MCQs) carrying 1 mark each.
3. Section B has 5 Short Answer-I (SA-I) type questions carrying 2 marks each.
4. Section C has 6 Short Answer-II (SA-II) type questions carrying 3 marks each.
5. Section D has 4 Long Answer (LA) type questions carrying 5 marks each.
6. Section E has 3 Case Based integrated units of assessment (4 marks each) with sub-parts of
the values of 1, 1 and 2 marks each respectively.
7. All Questions are compulsory. However, an internal choice in 2 Qs of 2 marks, 2 Qs of 3
marks and 2 Questions of 5 marks has been provided. An internal choice has been provided
in the 2 marks questions of Section E.
8. Draw neat figures wherever required. Take π =22/7 wherever required if not stated.

Section A

Section A consists of 20 questions of 1 mark each


S Marks
No
.
1. Which of the following is not a rational number? 1
(a)√6 (b)√9 (c)√25 (d)√36

2. 3 1
3 8 in decimal form is:
(a) 3.35 (b) 3.375 (c) 33.75 (d) 337.5
3. √3 is 1
a)a natural number (b) not a real number
(c) a rational number (d) an irrational number

4. The pair of equations x + 2y + 5 = 0 and –3x – 6y + 1 = 0 have 1


(a) infinite number of solutions (b) unique solution
(c) no solution (d) one solution
2
5. If the equation ax + bx + c = 0 has equal roots then c = 1
−𝑏 𝑏 −𝑏 2 𝑏2
(a) 2𝑎
(b) 2𝑎
(c) 4𝑎
(d) 4𝑎

6. Which of the following is a solution of the quadratic equation 2x2 + x – 6 = 1


0?
3
(a) x = -2 (b) x = –12 (c)2 (d) x = –3
7 The point P(1, 2) divides the join of A(–2, 1) and B(7, 4) are in the ratio of 1
(a) 3 : 2 (b) 2 : 3 (c) 2 : 1 (d) 1 : 2

8 The distance between the points A(2, –3) and B(2, 2) is 1


(a) 2 units (b) 4 units (c) 5 units (d) 3 units
9 (sec A + tan A) (1 – sin A) = 1
(a) sec A (b) sin A (c) cosec A (d) cos A

10 In a triangle PQR, right-angled at Q, PQ = 3 cm and PR = 6 cm, then 1


QPR =
(a) 0° (b) 30° (c) 45° (d) 60°
11 In the figure, if TP and TQ are the two tangents to a circle with centre O so 1
that POQ = 110°, then PTQ is equal to

(a) 600 (b) 700 (c) 800 (d) 900


12 The region between a chord and either of the arcs is called 1
(a) an arc (b) a Sector (c) a segment (d) a semicircle
13 The circumference of two circles are in the ratio 2 : 3. The ratio of their 1
areas is
(a) 2 : 3 (b) 4 : 9 (c) 9 : 4 (d) none of these
14 The difference between the circumference and the radius of a circle is 37 1
cm. Find the area of the circle.
(a) 111 cm2 (b) 184 cm2 (c) 154 cm2 (d) 259 cm2
15 The curved surface area of a sphere of radius 7 cm is: 1
(a) 516 cm2 (b) 616 cm2 (c) 716 cm2 (d) 880 cm2
16 1
The mean of the following frequency distribution is

C.I 0 – 10 10 – 20 20 – 30 30 – 40 40 – 50
F 12 16 6 7 9

(a) 12 (b) 16 (c) 22 (d) 20

17 If the mode of 12, 16, 19, 16, x, 12, 16, 19, 12 is 16, then the value of x is 1
(a) 12 (b) 16 (c) 19 (d) 18

18 A bag has 4 red balls and 2 yellow balls. A ball is drawn from the 1
bag without looking into the bag. What is probability of getting a
yellow ball?
1 2 1
(a)6 (b)3 (c)3 (d)1

Direction for questions 19 & 20: In question numbers 19 and 20, a


statement of Assertion (A) is followed by a statement of Reason (R).
Choose the correct option.
19 Assertion: If x = 2 sin 2 θ and y = 2 cos2 θ + 1 then the value of x + y = 3. 1
Reason : For any value of θ, sin 2 θ + cos2 θ = 1

(a) Both Assertion (A) and Reason (R) are true and Reason (R) is the
correct explanation of Assertion (A).
(b) Both Assertion (A) and Reason (R) are true but Reason (R) is not the
correct explanation of Assertion (A).
(c) Assertion (A) is true but Reason (R) is false.
(d) Assertion (A) is false but Reason (R) is true.
20 Assertion : D and E are points on the sides AB and AC respectively of a 1
ΔABC such that DE║BC then the value of x is 11, when AD = 4cm, DB =
(x – 4) cm, AE = 8cm and EC = (3x – 19) cm.
Reason : If a line divides any two sides of a triangle in the same ratio then
it is parallel to the third side.

(a) Both Assertion (A) and Reason (R) are true and Reason (R) is the
correct explanation
of Assertion (A).
(b) Both Assertion (A) and Reason (R) are true but Reason (R) is not the
correct explanation of Assertion (A).
(c) Assertion (A) is true but Reason (R) is false.
(d) Assertion (A) is false but Reason (R) is true.
Section-B
Section B consists of 5 questions of 2 marks each.
21 Sum of two numbers is 50 and their difference is 10, then find the numbers. 2

22 2
If cosθ = 0.6 , show that (5sinθ - 3tanθ)=0
OR
If A = 450, verify that
(i) sin2A = 2sinAcosA (ii) cos2A = 2cos2A – 1 = 1 – 2sin2A

23 𝐷𝐴 𝐴𝐸 2
In the given figure, ∠D=∠E and 𝐷𝐵 = 𝐸𝐶
. Prove that △BAC is isosceles triangle.
OR
Prove that if a line is drawn parallel to one side of a triangle to intersect the
other two sides in distinct points, the other two sides are divided in the
same ratio.

24 If all the sides of a parallelogram touch the circle, show that the 2
parallelogram is a rhombus.
25 A wire is looped in the form of a circle of radius 28 cm. It is re-bent into a 2
square form. Determine the length of the side of the square.

Section-C

26 There is a circular path around a sports field. Sonia takes 18 minutes to 3


drive one round of the field, while Ravi takes 12 minutes for the same.
Suppose they both start at the same point and at the same time, and go in
the same direction. After how many minutes will they meet again at the
starting point?

OR
Express each of the following positive integers as the product of its prime
factors: (i) 140 (ii) 156 (iii) 234
27 Find zeroes of following quadratic polynomial 48y2−13y−1 and verify 3
relationship between zeroes and its coefficients

28 The sum of ages of a man and his son is 45 years. Five years ago, the 3
product of their ages was four times the man’s age at that time. Find their
present ages.
OR
The difference of two numbers is 5 and the difference of their reciprocals is
1
. Find the numbers.
10

29 Prove that 𝑠𝑒𝑐 4 𝐴(1 − 𝑠𝑖𝑛4 𝐴) − 2 𝑡𝑎𝑛2 𝐴 = 1 3


30 PQR circumscribes a circle of radius r such that angle Q = 900 , PQ = 3 3
cm and QR = 4 cm. Find r.
31 Two coins are tossed simultaneously. Find the probability of getting 3
i). at least one head ii). at most one head iii). exactly
two head
Section-D
Section D consists of 4 questions of 5 marks each.
32 Solve the following system of linear equations graphically: 5
4x−y=4 ; 3x+2y=14
Shade the region bounded by the above lines and x-axis.

OR
Draw the graphs of the equations-
4x – y – 8=0; 2x - 3y + 6 = 0
Also determine the vertices ofthe triangle formed by the lines and x -
axis.
33 In the below figure, l || m and line segments AB, CD and EF are concurrent 5
𝐴𝐸 𝐴𝐶 𝐶𝐸
at point P. Prove that = =
𝐵𝐹 𝐵𝐷 𝐹𝐷

OR
A street light bulb is fixed on a pole 6 m above the level of street. If a woman of
height 1.5m casts a shadow of 3 m, find how far she is away from the base of the
pole?

34 5
A solid consisting of a right circular cone of height 120 cm and radius 60 cm
standing on a hemisphere of radius 60 cm is placed upright in a right circular
cylinder full of water such that it touches the bottom. Find the volume of water left
in the cylinder, if the radius of the cylinder is 60 cm and its height is 180 cm.

35 The mean of the following frequency distribution is 57.6 and the sum of the 5
observations is 50. Find f1 and f2
Class 0-20 20-40 40-60 60-80 80-100 100-120
Frequen 7 f1 12 f2 8 5
cy
Section- E
Case study based questions are compulsory.
36 Pratiksha was practicing focus and concentration technique by making a
card pyramid. Her pyramid included three cards in the first, i.e., topmost
layer, six cards in the second layer, 9 cards in the third layer and so on. Her
tower was 7 floor tall.
Answer the following questions using the above information.

I How many cards were required by her to make a 7 floor 1


tower?
II How many cards were required by her if she wanted to 1
make a 9 floor tower?
III What is the difference between the number of card required 2
between 2nd floor and 11th floor if she made a 11 floor
tower?
OR
Pratiksha built a 7 floor card tower. How many more cards
does she need to make a 10 floor tower?
37
Cartesian coordinate system is considered inside the area of a garden to
understand the heights, distances and various other mathematical
measurement parameters. The above positions marked are for the plants
grown in the garden.
Answer the following questions using the above information.

1 Find co-ordinates of Lily. 1


2 Are points of Rose, Lily and Marigold collinear? 1
3 What is the distance between Rose and Jasmine? 2
Or
Find co-ordinates of point which which divides line segment
between points of Hibiscus and Dahlia in 1:1.

38

B A

Vyom on his way to office came across a lamp post at a distance of 20cm
from him. He observer that angle of elevation to top of lamp post is 450
from top of his head. Vyom’s height is 100cm.

Answer the following questions using the above information.

(I) What is the height of the lamp post ? 1


(II) What is the value of ∠𝐴𝐶𝐵 1
(III What is the length of the line of sight of Vyom to the top of 2
) the lamp post ?
OR
0
If angle of elevation is 60 what will be Vyom’s distance from
the lamp post?
KENDRIYA VIDYALYA SANGATHAN, JAMMU REGION
Sample Paper Set - 6

CLASS – X SUBJECT -Mathematics


Basic (241)
Time : 3 Hours MM:
80
General Instructions:
1. This Question Paper has 5 Sections A,B,C.D and E.
2. Section A has 20 MCQs carrying 1 mark each
3. Section B has 5 questions carrying 02 marks each.
4. Section C has 6 questions carrying 03 marks each.
5. Section D has 4 questions carrying 05 marks each.
6. Section E has 3 case based integrated units of assessment (04 marks each) with sub-
parts of the values of 1, 1 and 2 marks each respectively.
7. All Questions are compulsory. However, an internal choice in 2 Qs of 5 marks, 2 Qs of
3 marks and 2 Questions of 2 marks has been provided. An internal choice has been
provided in the 2 marks questions of Section E
8. Draw neat figures wherever required. Take π =22/7 wherever required if not stated.
SECTION A

Section A consists of 20 questions of 1 mark each.


SR MARKS
NO
1 If two positive integers a and b are written as a = x3y2 and b = xy3: where x,y are prime 1
numbers, then HCF of a and b is
(a) xy2 (b) x2 y2 (c) x3 y2 (d) x4 y2
2 What is the exponent of 3 in the prime factorization of 162. 1
a. 2
b. 0
c. 4
d. 1
3 The pair of linear equations 5x – 3y = 11 and – 10x + 6y = – 22 are 1
a. dependent(consistent)
b. None of these
c. consistent
d. inconsistent
4 A quadratic equation ax2+bx + c= 0 has real and distinct roots, if 1
a.b2-4ac > 0
b. b2-4ac < 0
c. None of these
d. b2-4ac = 0
5 Rohan’s mother is 26 years older than him. The product of their ages 3 years from now 1
will be 360, then Rohan’s present age is
a. 6 years
b. 7 years
c. 10 years
d. 8 years
6 If one end of a diameter of a circle is (4, 6) and the centre is ( – 4, 7), then the other end 1
is
a. ( – 12, 8)
b. (8, – 12)
c. (8, 10)
d. (8, – 6)
7 If tan2 θ = √𝟑 , then the value of θ is 1
a. 60˚
b. 45 ˚
c. 0˚
d. 30˚
8 If sinA +1 sin2A = 1 , then value of cos2A + cos4A is 1
1. – 1
2. 2
3. 0
4. 1
9 The marks obtained by 9 students in Mathematics are 59, 46, 30, 23, 27, 44, 52, 40 and 1
29. The median of the data is
a. 35
b. 29
c. 30
d. 40
10 1 1
If sin A = , the value of cot A is
2
a. √3
1
b.
√5
c. 1
1
d.
√3
11 A street light is fixed on a pole 6 m above the ground. If a woman of height 1.5 m casts 1
a shadow of 3, then distance between her and the base of the pole is
a. 12 m
b. 9 m
c. 8 m
d. 10 m
12 In the given figure, if OQ = 3cm, OP= 5m, then the length of 1
PR is
a. 4 cm
b. 3 cm
c. 5 cm
d. 6 cm
13 A tangent to a circle is line that intersects the circle at 1
1. two points
2. three points
3. one point only
4. none of the above
14 The coordinates of origin are 1
a. (0,x)
b. (y,0)
c. (0,0)
d. (x,y)
15 The volume of the cuboid whose length, breadth and height is 12cm, 8cm and 6cm is 1
a. 568 cu.cm
b. 576 cu.cm
c. 570 cu.cm
d. 576 sq.cm
16 A bicycle wheel makes 5000 revolutions in moving 11km. The diameter of the wheel is 1
a. 100cm
b. 35cm
c. 140cm
d. 70cm
17 What is the empirical relation between the three measures of central tendency is 1
(a) 3 Median = 2 Mean + Mode
(b) 2 Median = 3Mean + Mode
(c) 3 Median = 2 Mean + 2 Mode
(d) Median = 2 Mean + Mode
18 If P(E) = 0.05, what will be the probability of 'not E'? 1
a. 0.55
b. 0.59
c. 0.95
d. 0.095
19 Assertion : The HCF of two numbers is 18 and their product is 3072. Then their 1
LCM = 169.
Reason : If a, b are two positive integers, then HCF x LCM = a x b.
(a)Both assertion (A) and reason (R) are true and reason (R) is the correct explanation
of assertion (A).
(b)Both assertion (A) and reason (R) are true but reason (R) is not the correct
explanation of assertion (A).
(c)Assertion (A) is true but reason (R) is false.
(d)Assertion (A) is false but reason (R) is true.
20 Assertion : The point on the X -axis which if equidistant from the points A(- 2, 3) and 1
B(5, 4) is (2, 0)
Reason : The coordinates of the point P(x, y) which divides the line segment joining the
𝑚1𝑥2 + 𝑚2 𝑥1 𝑚1𝑦2 + 𝑚2 𝑦1
points A(x1 , y1 ) and B(x2 , y2 ) in the ratio m1 : m2 is ( 𝑚1 + 𝑚2
, )
𝑚1 + 𝑚2
(a)Both assertion (A) and reason (R) are true and reason (R) is the correct explanation
of assertion (A).
(b)Both assertion (A) and reason (R) are true but reason (R) is not the correct
explanation of assertion (A).
(c)Assertion (A) is true but reason (R) is false.
(d)Assertion (A) is false but reason (R) is true.
SECTION B
Section B consists of 5 questions of 2 marks each.
21 find the value of k for which the system of equations 3x + ky = 0, 2x - y = 0 has a 2
unique solution.
22 If θ = 30°, verify that cos 3 θ = 4cos3 θ - 3cos θ . 2
23 In △ ABC, P and Q are points on sides AB and AC respectively PQ II BC, such that If 2
AP = 4 cm, PB = 6 cm and PQ = 3 cm, determine BC.

24 In the given figure, two circles touch each other at the point C. Prove that the common 2
tangent to the circles at C, bisects the common tangent at P and Q.

OR

In the given figure, TP and TQ are tangents from T to the


circle with centre O and R is any point on the circle. If AB is
a tangent to the circle at R, prove that TA + AR = TB + BR.

25 A garden roller has a circumference of 4 m. Find the number of revolutions, it makes in 2


moving 40 m.
OR
A rope by which a cow is tethered is increased from 16m to 23m. How much additional
ground does it have now to graze?
SECTION C
Section C consists of 6 questions of 3 marks each.
26 Prove that √3 is irrational number. 3

27 If the zeroes of the polynomial x2 + px + q are double in value to the zeroes of 3


2x2 – 5x – 3, find the value of p and q.
28 In a class test, the sum of Shefali's marks in Mathematics and English is 30. Had she 3
got 2 marks more in Mathematics and 3 marks less in English, the product of their
marks would have been 210. Find her marks in the two subjects.
1−𝑐𝑜𝑠𝛉 3
29 𝑃𝑟𝑜𝑣𝑒 𝑡ℎ𝑎𝑡 (𝑐𝑜𝑠𝑒𝑐𝛉 − 𝐜𝐨𝐭𝛉)2 =
1+𝑐𝑜𝑠𝛉
OR
𝑐𝑜𝑠𝐴 1+𝑠𝑖𝑛𝐴
1+𝑠𝑖𝑛𝐴
+ 𝑐𝑜𝑠𝐴 = 2 sec A

30 In the given figure, O is the centre of a circle. PT and PQ are tangents to the circle
from an external point P. If ∠TPQ = 70°, find ∠TRQ .

OR
Two concentric circles are of radii 5 cm and 3 cm. Find the length of the chord of the
larger circle which touches the smaller circle.
31 All red face cards are removed from a pack of playing cards. The remaining cards are
well-shuffled and then a card is drawn at random from them. Find the probability
that the drawn card is
i. a red card,
ii. a face card,
iii. a card of clubs.
SECTION D
Section D consists of 4 questions of 5 marks each.
32 The taxi charges in a city consist of a fixed charge together with the charge for the 5
distance covered. For a distance of 10 km, the charge paid is ₹105 and for a journey of
15 km, the charge paid is ₹155. What are the fixed charges and the charges per km?
How much does a person have to pay for travelling a distance of 25 km?
OR
A train covered a certain distance at a uniform speed. If the train would have been 10
km/h faster, it would have taken 2 hours less than the scheduled time. And, if the train
were slower by 10 km/h, it would have taken 3 hours more than the scheduled time.
Find the distance covered by the train.

33 5
State and prove Basic Proportionality theorem.

34 A solid toy is in the form of a 5


hemisphere surmounted by a right circular cone. The
height of the cone is 2 cm and the diameter of the
base is 4 cm. Determine the volume of the toy. If a
right circular cylinder circumscribes the toy, find the
difference of the volumes of the cylinder and the toy.

OR
A juice seller was serving his customers using glasses as shown in Fig.
The inner diameter of the cylindrical glass was 5 cm, but the bottom of
the glass had a hemispherical raised portion which reduced the
capacity of the glass. If the height of a glass was 10 cm, find the
apparent capacity of the glass and its actual capacity. (Use π= 3.14.)

35 100 surnames were randomly picked up from a local telephone directory and the 5
frequency distribution of the number of letters in the English alphabet in the surnames
was obtained as follows:
Number 1-4 4-7 7-10 10-13 13-16 16-19
of letters
Number 6 30 40 16 4 4
of
surnames
Determine the median number of letters in the surnames. Find the mean number of
letters in the surnames. Also, find the modal size of the surnames.
SECTION E
Case study based questions are compulsory.
36 CASE STUDY – 1 4

In a school, students thought of planting trees in and around the school to reduce air
pollution. It was decided that each section of each class would plant twice as many
plants as the class standard. There were 3 sections of each standard from 1 to 12. So, if
there are three sections in class 1 say 1A, 1B and 1C, then each section would plant 2
trees. Similarly, each section of class 2 would plant 4 trees and so on. Thus, the number
of trees planted by classes 1 to 12 formed an AP given by 6, 12, 18,...

Based on above information answer the following question:

a) What is the common difference of the AP formed? 1

b) What will be the nth term of the AP formed? 1

c) What will be the third term from the end of the AP formed?
OR 2
How many trees will be planted by the students of all the sections of class 8?
37 CASE STUDY-2
Students of a school are standing in rows and columns in their playground for a drill
practice. A, B, C and D are the positions of four students as shown in the figure.
Based on above information answer the following question:

a) Is it possible to place Ram(R) in the drill in such a way that he is equidistant from all 1
the four students A, B, C and D? (1)

b) What are the coordinates of the position of Ram? 1


c) What are the coordinates of C and D respectively?
OR 2
What are the coordinates of A and B respectively?

38 CASE STUDY 3 4

A group of students of class X visited India Gate on an education trip. The teacher and
students had interest in history as well. The teacher narrated that India Gate, official
name Delhi Memorial, originally called All-India War Memorial, monumental
sandstone arch in New Delhi, dedicated to the troops of British India who died in wars
fought between 1914 and 1919.The teacher also said that India Gate, which is located at
the eastern end of the Rajpath (formerly called the Kingsway), is about 138 feet (42
metres) in height.

Based on above information answer the following questions:


a) 1

The angle formed by the line of sight with the horizontal line when the object viewed is
below the horizontal level is called?
b) What is the value of tan 60° ? 1

c) What is the angle of elevation if they are standing at a distance of 42m away from the 2
monument?
OR
The ratio of the length of a rod and its shadow is 1:1 . What is the angle of elevation of
the Sun ?
KENDRIYA VIDYALYA SANGATHAN, JAMMU REGION
Sample Paper Set - 7

CLASS – X SUBJECT -Mathematics Basic (241)


Time : 3 Hours MM: 80
General Instructions:
1. This Question Paper has 5 Sections A-E.
2. Section A has 20 MCQs carrying 1 mark each
3. Section B has 5 questions carrying 02 marks each.
4. Section C has 6 questions carrying 03 marks each.
5. Section D has 4 questions carrying 05 marks each.
6. Section E has 3 case based integrated units of assessment (04 marks each) with subparts of the
values of 1, 1 and 2 marks each respectively.
7. All Questions are compulsory. However, an internal choice in 2 Qs of 5 marks, 2 Qs of 3 marks and
2 Questions of 2 marks has been provided. An internal choice has been provided in the 2marks
questions of Section E
8. Draw neat figures wherever required. Take π =22/7 wherever required if not stated.
SECTION A
S. MARKS
NO.
Q.1:- Express 98 as a product of its primes 1
(a) 2² × 7 (b) 2² × 7²
(c) 2 × 7² (d) 23 × 7
Q.2:- If two positive integers A and B can be expressed as A = xy3 and B = x4y2z; x, y being 1
prime numbers then HCF (A, B) is
(a) xy² (b) x4y²z
(c) x4y3 (d) x4y3z
Q.3:- The zeroes of x2–2x –8 are: 1
(a) (2,-4) (b) (4,-2)
(c) (-2,-2) (d) (-4,-4)
Q.4:- The value of ‘k’ for which the system of equation 2x + 3y = 5 and 4x + ky = 10 has 1
infinite number of solutions is

(a) k = 1 (b) k = 3
(c) k = 6 (d) k = 0
2
Q.5:- If the roots of px + qx + 2 = 0 are reciprocal of each other, then 1
(a) p = 0 (b) p = -2
(c) p = ±2 (d) p = 2
Q.6:- The distance of the point P (–6, 8) from the origin is 1
(a) 8 (b) 27
(c) 10 (d) 6
Q.7:- In the given figure, ΔACB ~ ΔAPQ. If AB = 6 cm, BC = 8 cm, and PQ = 4 cm then AQ is 1
equal to

(a) 2 cm (b) 2.5 cm


(c) 3 cm (d) 3.5 cm
Q.8:- Find the point on X-axis which is equidistant from the points (-2, 5) and (2, -3). 1
(a) (1, 1) (b) (1, 0)
(c) (-2, 0) (d) (3, 0)
Q.9:- The distance between two parallel tangents of a circle of radius 4 cm is 1
(a) 2 cm (b) 4 cm
(c) 6 cm (d) 8 cm
2 0 2 0
Q.10:- The value of the (𝑠𝑖𝑛 45 + 𝑡𝑎𝑛 60 ) is -------------------- 1
7 2
(a) (b)
2 7
3 3
(c) (d)
2 4
Q.11:- If ∆ABC is right angled at C , then the value of sec(A + B) is 1
(a) 1 (b) 0
2 (d) not defined
(c)
√3
Q.12:- If length of the shadow and height of a tower are in the ratio 1:1. Then find the angle of 1
elevation.

(a) 00 (b) 450


(c) 900 (d) not defined
Q.13:- In the given figure, if ∠RAS = 25°, the value of ∠ROS is 1

(a) 135° (b) 145°


(c) 165° (d) 155°
Q.14:- A tangent is drawn from a point at a distance of 17 cm of circle C(0, r) of radius 8 cm. The 1
length of its tangent is
(a) 5 cm (b) 9 cm
(c) 15 cm (d)23 cm
3
Q.15:- The volume of a sphere is 524 cm . The diameter of sphere is 1
(a) 5cm (b) 4cm
(c) 3 cm (d) 10 cm
Q.16:- Relation between mean, mode and median is: 1
(a) Mean =3median-2 mode (b) median=3 mode-2mean
(c) mode =3median-2mean (d) mode =3median+2mode
Q.17:- If the sum of 16 observations is 480+y and mean of the observation is y. Then value of y 1
is:
(a) 30 (b) 24
(c) 27 (d) 32
Q.18:- If P (E) = 0.07, then what is the probability of ‘not E’? 1
(a) 0.93 (b) 0.95
(c) 0.89 (d) 0.90
Assertion and Reason based questions
DIRECTION: In the question number 19 and 20, a statement of assertion (A) is followed
by a statement of Reason (R). Choose the correct option Statement

Q.19:- Statement (A):- HCF (23, 53) is 1. 1


Reason (R): If p and q are prime then HCF (p, q) = 1

(a) Both assertion (A) and reason (R) are true and reason (R) is the correct explanation
of assertion (A)
(b) Both assertion (A) and reason (R) are true and reason (R) is not the correct
explanation of assertion (A)
(c) Assertion (A) is true but reason (R) is false
(d) Assertion (A) is false but reason (R) is true.
Q.20:- Statement (A):- The value of y is 6, for which the distance between the points P 1
(2, − 3) and Q (10, y) is 10

Reason (R): Distance between two given points A (𝑥1 , 𝑦1 ) and B (𝑥2 , 𝑦2 ) is given ,
AB =√(𝑥2 −𝑥1 )2 + (𝑦2 −𝑦1 )2
(a) Both assertion (A) and reason (R) are true and reason (R) is the correct explanation
of assertion (A)
(b) Both assertion (A) and reason (R) are true and reason (R) is not the correct
explanation of assertion (A)
(c) Assertion (A) is true but reason (R) is false
(d) Assertion (A) is false but reason (R) is true.
SECTION B
Section B consists of 5 questions of 2 marks each.
Q.21:- Check whether the following pair of linear equations is consistent or inconsistent. 2
2x + y – 6 = 0
4x – 2y – 4 = 0

Q.22:- In the figure if PQ∥RS, prove that △POQ∼△SOR. 2

OR

Observe Fig. and


then find ∠P.

Q.23:- What will be the area of a quadrant of a circle given that its circumference is 42 cm. (Take 2
𝜋 = 3.14)
Q.24:- If sin A = cos A, find the value of 2tan²A + 2 sin²A + 1. 2
Q.25:- 2
The circumference of two circles is in the ratio of 2:3. Find the ratio of their areas.
OR
The radii of two circles are 8 cm and 6 cm respectively. Find the radius of the circle
having its area equal to the sum of the areas of the two circles.
SECTION C
Section C consists of 6 questions of 3 marks each.
Q.26:- Prove that √7 is irrational. 3
Q.27:- Find a quadratic polynomial whose zeros are 5+√2 and 5−√2 3
Q.28:- Solve 2x + 3y = 11 and 2x – 4y = – 24 and hence find the value of ‘m’ for which y = mx + 3
3.
OR
9
A fraction becomes , if 2 is added to both the numerator and the denominator. If, 3 is
11
added to both the numerator and the denominator it becomes 5/6. Find the fraction.
Q.29:- In the given figure, PA and PB are tangents to the circle with centre O. If ∠APB = 60°, 3
then calculate ∠OAB.

Q.30:- If cos θ + sin θ=√2 cos θ, To show that cos θ – sin θ=√2 sin θ . 3
OR
4sinA−cos A+1
If 4 tan A = 3, evaluate
4 sinA+cos A−1
Q.31:- 1. Two dice are numbered 1,2,3,4,5,6 and 1,2,2,3,3,4 respectively. They are thrown 3
And the sum of numbers on them is noted. Find the probability of getting
i) Sum 7 ii) sum is perfect square

SECTION D
Section D consists of 4 questions of 5 marks each.
Q.32:- A motor boat whose speed is 20 km/h in still water, takes 1 hour more to go 48 km 5
upstream than to return downstream to the same spot. Find the speed of the stream.

OR
A rectangular park is to be designed whose breadth is 3 m less than its length. Its area is to
be 4 square meters more than the area of a park that has already been made in the shape of
an isosceles triangle with its base as the breadth of the rectangular park and of altitude 12
m. Find the length and breadth of the rectangular park.
Q.33:- State and prove the Thales theorem. 5
Q.34:- A toy is in the form of a cone of radius 3.5 cm mounted on a hemisphere of the same 5
radius. The total height of the toy is 15.5 cm. Find the total surface area of the toy.
OR
Rachel, an engineering student, was asked to make a model shaped like a cylinder with
two cones attached at its two ends by using a thin aluminum sheet. The diameter of the
model is 3 cm and its length is 12 cm. If each cone has a height of 2 cm, find the volume
of air contained in the model that Rachel made. (Assume the outer and inner dimensions
of the model to be nearly the same)
Q.35:- The following distribution gives the state-wise teacher- student ratio in higher secondary 5
schools of India. Find the mode and mean of this data. Interpret the two measures.
SECTION E
Case study based questions are compulsory
Q.36:- Ravi’s father had to undergo knee surgery as he was suffering
from lot of pain and unable to walk properly. The physiotherapist
asked him to return to his jogging program slowly. He suggested
jogging for 10 minutes daily for first week thereafter increase the
jogging time by 6 minutes.
1
(i) Find the AP formed by the number giving information about
the time taken for jogging by Ravi’s father.

(ii) What will be his jogging time 8 weeks? 1

(iii) In which week will his jogging time be 70 minutes per day?
OR
2
Which term of the AP: 50, 47, 44,……… is the first
negative term?

Q.37:- Diagram given below show the front view and side view of a hut.
(i) Find the coordinates of the point C.
(ii) Referring to the point find the distance between the points A and D
(iii) Find coordinates of the point that divides the line segment joining the points P
and S in the ration 1: 3. 1
Or 1
Refer to the side view if a point (x, y) is equidistant from the point Q and S
then find the relation between x and y.
2
Q.38:- Rohit is standing at the top of the building observe a car at an angle 30° which is
approaching the foot of building at a uniform speed. 6 second later, the angle of
depression of car formed to be 60°, whose distance at that instant from the building is
25m.

(i) Find the height of the building. 1


(ii) What kind of change happens in angle of elevation when point of observation 1
moves towards the object.
(iii) Find the distance between two positions of the car. 2
OR
Find the total time taken by the car to reach the foot of the building from
starting point.

******************************************************************
KENDRIYA VIDYALYA SANGATHAN, JAMMU REGION
Sample Paper Set - 8

CLASS – X SUBJECT -Mathematics


Basic (241)
Time : 3 Hours MM:
80
General Instructions:

1. This Question Paper has 5 Sections A-E.


2. Section A has 20 MCQs carrying 1 mark each
3. Section B has 5 questions carrying 02 marks each.
4. Section C has 6 questions carrying 03 marks each.
5. Section D has 4 questions carrying 05 marks each.
6. Section E has 3 case based integrated units of assessment (04 marks each) with subparts of the
values of 1, 1 and 2 marks each respectively.
7. All Questions are compulsory. However, an internal choice in 2 Qs of 5 marks, 2 Qs of 3
marks and 2 Questions of 2 marks has been provided. An internal choice has been provided in
the 2marks questions of Section E
8. Draw neat figures wherever required. Take π =22/7 wherever required if not stated.

SECTION A
Section A consists of 20 questions of 1 mark each.
S. MA
NO RKS

1 HCF of p and q if p = ab and q = xy 1


(a) 1 (b) ax(C) by(d) abxy
2 A sweetseller has 420 kajubarfis and 130 badambarfis. She wants to stack them in such a way that 1
each stack has the same number, and they take up the least area of the tray. What is the number of that
can be placed in each stack for this purpose
(a) 10(b) 20(c)30(d)5
3 The one zero of the polynomial x2 – 3 is 1
(a) − 3(b) +3(c)√3(d)0
4 x – 2y = 0 , 3x + 4y – 20 = 0Algebraic interpretation of the given equation is 1
(a) unique solution (b) Infinitely many solution (c) no solution (d) none of these
5 If 1/2 is a root of the equation x2 + kx – 5/4 = 0, then the value of k is 1
(a) 2 (b) -2 (c) 14 (d)12
6 If AOBC is a rectangle whose three vertices are A(0, 3), O(0, 0) and B(5, 0), then the length of its 1
diagonal is
(a) 5 (b) 3 (c) √34 (d) 4
7 If in two triangles ABC and PQR, PQ/CA =QR/AB = PR/CB then 1
(a) ∆PQR ~ ∆CAB (b) ∆PQR ~ ∆ABC (c) ∆CBA ~ ∆PQR (d) ∆BCA ~ ∆PQR
8 The coordinates of the point which divides the line segment joining the points (4, – 3) and (8, 5) in the 1
ratio 3 : 1 internally
(a) 7,3 (b) 3,7 (C) 5,2 (d) 2,5
9 In figure, AB is a chord of the circle and AOC is its diameter such that ∠ACB = 50°. If AT is the 1
tangent to the circle at the point A, then ∠BAT is equal to

(a) 65° (b) 60° (c) 50° (d) 40°

10 A pole 6m high casts a shadow 2√3 m long on the ground, then the Sun’s elevation is 1
(a) 600 (b) 45° (C) 30° (d) 90°
11 Given that sinθ = a / b, then cosθ is equal to 1

12 If 5tan A= 3 , then the value cot A is 1


(a) 3/5 (b) 5/3 (c) 25/9 (d) 9/25
13 If the perimeter and the area of a circle are numerically equal, then the radius of the circle is 1
(a) 2 units (b) 6 units (c) 4 units (d) 7 units
14 The area of the circle inscribed in a square of side a cm is 1
(a) πa2 sq units (b) πa2 / 4 sq units (c) a2sq units (d) π/4 sq units

15 If two solid hemispheres of same base radius r are joined together along their bases, then curved 1
surface area of this new solid is
(a) 4πr2 (b) 6πr2 (c) 3πr2 (d) 8πr2
16 The mean of following distribution is: 1
Xi 11 14 17 20
fi 3 6 8 7

(a) 15.6 (b) 17 (c) 14.8 (d) 16.4

17 The mode and mean is given by 7 and 8, respectively. Then the median is: 1
(a) 1/13 (b) 13/3 (c) 23/3 (d) 33
18 If P (A) denotes the probability of an event A, then 1
(a) P(A) < 0 (b) P(A) > 1 (c) 0 ≤ P(A) ≤ 1 (d) -1 ≤ P(A) ≤ 1
DIRECTION: In the question number 19 and 20, a statement of assertion (A) is followed by a 1
statement of Reason (R). Choose the correct option
19 Statement A (Assertion): A number N when divided by 15 gives the remainder 2. Then the
remainder is same when N is divided by 5.
Statement R (Reason) : √3 is an irrational number.
(a) Both Assertion (A) and Reason (R) are true and Reason (R) is the correct explanation of Assertion
(A).
(b) Both Assertion (A) and Reason (R) are true but Reason (R) is not the correct explanation of
Assertion (A).
(c) Assertion (A) is true but Reason (R) is false.
(d) Assertion (A) is false but Reason (R) is true.
20 Statement A (Assertion): The points (- 4, 0), (4, 0) and (0, 3) are the vertices of a isosceles triangle 1
Statement R (Reason) : An isosceles triangle has two sides equal.
(a) Both Assertion (A) and Reason (R) are true and Reason (R) is the correct explanation of Assertion
(A).
(b) Both Assertion (A) and Reason (R) are true but Reason (R) is not the correct explanation of
Assertion (A).
(c) Assertion (A) is true but Reason (R) is false.
(d) Assertion (A) is false but Reason (R) is true.
SECTION B
Section B consists of 5 question of 2 marks each
21 Solve for x and y :- x-y+1 = 0 ; 4x +3y -10 =0 2

22 . 2

If a line intersects sides AB and AC of a ∆ ABC at D and E respectively and is parallel to BC, prove
that AD /AB = AE /AC figure given below.

OR
In Fig. given below PS/ SQ = PT/ TR and ∠ PST = ∠ PRQ. Prove that PQR is an isosceles triangle.

23 To prove The tangent at any point of a circle is perpendicular to the radius through the point of 2
contact.
24 If sinθ – cosθ = 0, then the value of (sin4θ + cos4θ) ? 2

25 Find the area of a sector of a circle of radius 28 cm and central angle 45°. 2
OR
The wheel of a motor cycle is of radius 35 cm. How many revolutions per minute must the wheel
make, so as to keep a speed of 66 km/h?
SECTION C
Section C consists of 6 question of 3 marks each
26 To prove that √3 is irrational 3
27 If α and β are the zeros of x2 + 7x + 12 then find the value of 1/α + 1/ β - 2αβ 3
28 A two-digit number is obtained by either multiplying the sum of the digits by 8 and then subtracting 5 3
or by multiplying the difference of the digits by 16 and then adding 3. Find the number.
OR
Vijay had some bananas and he divided them into two lots A and B. He sold the first lot at the rate of
₹ 2 for 3 bananas and the second lot at the rate of ₹ 1 per banana and got a total of ₹ 400 If he had sold
the first lot at the rate of ₹ 1 per banana and the second lot at the rate of ₹ 4 for 5 bananas, his total
collection would have been ₹ 460. Find the total number of bananas he had.
29 A quadrilateral ABCD is drawn to circumscribe a circle . Prove that AB + CD = AD + BC 3

30 To Prove that 3
sec A (1 – sin A)(sec A + tan A) = 1.
OR
(cot A – cos A)/ (cot A + cos A) = (cosec A – 1)/( cosec A + 1)

31 Two dice are thrown together. Find the probability that the product of the numbers on the top of the 3
dice is
(i) 6
(ii) 12
(iii) 7
SECTION D
Section D consists of 4 question of 5 marks each
32 A train travels at a certain average speed for a distance of 63 km and then travels a distance of 72 km 5
at an average speed of 6 km/h more than its original speed. If it takes 3 hours to complete the total
journey, what is its original average speed?
OR

A train, travelling at a uniform speed for 360 km, would have taken 48 minutes less to travel the same
distance if its speed were 5 km/h more. Find the original speed of the train
33 In figure given below A, B and C are points on OP, OQ and OR respectively such that AB || PQ and AC || PR. 5
Show that BC || QR
.
34 The decorative block shown in Fig given is made of two solids — a cube and a hemisphere. The base of the 5
block is a cube with edge 5 cm, and the hemisphere fixed on the top has a diameter of 4.2 cm. Find the total
surface area of the block

OR
Rasheed got a playing top (lattu) as his birthday present, which surprisingly had no colour on it. He wanted to
colour it with his crayons. The top is shaped like a cone surmounted by a hemisphere (see Fig 13.6). The entire
top is 5 cm in height and the diameter of the top is 3.5 cm. Find the area he has to colour. (Take π = 22 /7 )

35 The median of the following data is 525. Find the values of x and y, if the total frequency is 100. 5

Height(in frequency
cm)
0-100 2
100-200 5
200-300 X
300-400 12
400-500 17
500-600 20
600-700 Y
700-800 9
800-900 7
900-1000 4
100

SECTION E
Case study based question are compulsory
36 Kanika was given her pocket money on Jan 1st, 2008. She puts ₹ 1 on Day 1, ₹ 2 on Day 2, ₹ 3 on
Day 3, and continued doing so till the end of the month, from this money into her piggy bank. She
also spent ₹ 204 of her pocket money and found that the end of the month she still had ₹ 100 with her.

1
i) What is the first term and common difference?
ii) What is the sum of money spent and money left with Kanika ? 1
iii) How much was her pocket money for the month?
OR 2
How much money she has in her piggy bank?

37

SCHOOL
BANK

HOUS OFFICE
E

Mr. Tom starts walking from his house to office. Instead of going to the office directly, he goes to a
bank first, from there to his daughter’s school and then reaches the office.
(Assume that all distance covered are in straight lines). If the house is situated at (2, 4), bank at (5, 8),
school at (13,14) and office at (13, 26) and coordinates are in km.

i) What is the distance between house to bank ? 1


1
ii) What is the shortest distance between house to office?
iii) What is the distance travelled by Ayush in reaching his office?
OR 2
What is the extra What is the distance travelled by Ayush in reaching his office?

38
A group of students of class X visited Qutb Minar on an education trip. The teacher and students had
interest in history as well. The Qutb Minar, also spelled Qutub Minar and Qutab Minar, is
a minaret and "victory tower" that forms part of the Qutb complex, which lies at the site of Delhi’s
oldest fortified city, Lal Kot, founded by the Tomar Rajputs.[3] It is a UNESCO World Heritage
Site in the Mehrauli area of South Delhi, India.[4][5] It is one of the most visited tourist spots in the
city, mostly built between 1199 and 1220. 62 metres in height.

i) What is the angle of elevation if they are standing at a distance of 62m away from the 1
monument?
ii) The ratio of the length of a rod and its shadow is 1:1 . The angle of elevation of the Sun is 1
iii) They want to see the tower at an angle of 60 . So, they want to know the distance where they 2
should stand and hence find the distance.

OR
If the altitude of the Sun is at 60 , then the height of the vertical tower that will cast a shadow of
length 20 m is
KENDRIYA VIDYALYA SANGATHAN, JAMMU REGION
Sample Paper Set - 9

CLASS – X SUBJECT -Mathematics Basic


(241)
Time : 3 Hours MM: 80

General Instructions:
1. This Question Paper has 5 Sections A-E.
2. Section A has 20 MCQs carrying 1 mark each
3. Section B has 5 questions carrying 02 marks each.
4. Section C has 6 questions carrying 03 marks each.
5. Section D has 4 questions carrying 05 marks each.
6. Section E has 3 case based integrated units of assessment (04 marks each) with subparts of the values of
1, 1 and 2 marks each respectively.
7. All Questions are compulsory. However, an internal choice in 2 Qs of 5 marks, 2 Qs of 3 marks and 2
Questions of 2 marks has been provided. An internal choice has been provided in the 2marks questions
of Section E
8. Draw neat figures wherever required. Take π =22/7 wherever required if not stated.

SECTION- A
1.The smallest number by which √𝟐𝟕 should be multiplied so as to get a rational number

(a) √𝟐𝟕 (b) 𝟑√𝟑 (c) √𝟑 (d) 3


2. The HCF of (26,169 ) = 13, then LCM ( 26, 169 ) =

(a) 26 (b) 52 (c) 338 (d) 13

3.What should be added to the polynomial 𝐱 𝟐 – 𝟓𝐱 + 𝟒,so that 3 is the zero of the resulting
polynomial

(a) 1 (b) 2 (c) 4 (d) 5

4. The pair of linear equations 2kx + 5y = 7, 6x-5y= 11 has a unique solution, if

𝟐 𝟐
(a) k ≠ -3 (b) k≠𝟑 (c) k ≠ 5 (d) k ≠ 𝟗
5.The distance between (4, 4) and (1,0) is:
(A) 15 units (B) 5 units C) 25 units (D) 41 units

6. A fair die is thrown once. The probability of getting a composite number less than 5 is
𝟏 𝟏 𝟐
(a) 𝟑
(b) 𝟔 (c)𝟑(d) 0
7. For which value(s ) of p, will the lines represented by the following pair of linear equations be
parallel

3x-y-5 =0

6x-2y-p =0

(a) all real values except 10 (b) 10 (c) 5/2 (d) ½

8. Number of zeroes in the given polynomial

a) 1 (b) 0 (c) 2 (d) 3

9.The point which divides the line segment of points P(-1, 7) and (4, -3) in the ratio of 2:3 is:
(a) (-1, 3) (b) (-1, -3) (c) (1, -3) (d) (1, 3)

10. If the distance between the points (x, -1) and (3, 2) is 5, then the value of x is
(a) -7 or -1
(b) -7 or 1
(c) 7 or 1
(d) 7 or -1
11. Ratios of sides of a right triangle with respect to its acute angles are known as
(a) trigonometric identities
(b) trigonometry
(c) trigonometric ratios of the angles
(d) none of these
12. (Sin 30°+cos 60°)-(sin 60° + cos 30°) is equal to:
(a) 0
(b) 1+2√3
(c) 1-√3
(d) 1+√3
13. A line intersecting a circle in two points is called a _______.
(a) Secant
(b) Chord
(c) Diameter
(d) Tangent
14. The area of a sector of a circle with radius 6 cm if the angle of the sector is 60°.
(a) 142/7
(b) 152/7
(c) 132/7
(d) 122/7
15. In a circle of radius 21 cm, an arc subtends an angle of 60° at the centre. The length of the arc
is;
(a) 20cm
(b) 21cm
(c) 22cm
(d) 25cm
16. The total surface area of a solid hemisphere of radius r is:
(a) 47πr²
(b) 2πr²
(c) 43πr³
(d) 3πr²
17.The mode and mean is given by 7 and 8, respectively. Then the median is:
(a) 1/13
(b) 13/3
(c) 23/3
(d) 33
18.The mean of the data: 4, 10, 5, 9, 12 is;
(a) 8
(b) 10
(c) 9
(d) 15
19. Assertion : The HCF of two numbers is 5 and their productis 150, then their LCM
is 30
Reason : For any two positive integers a and b,HCF (a,b)+LCM (a,b) = a x b .
(a) Both assertion and reason are true and reason is the correct explanation of assertion
(b) Both assertion and reason are true but reason is not the correct explanation of
assertion .
(c) Assertion is true but reason is false.
(d) Assertion is false but reason is true

𝟒
20.Assertion : The value of sinθ = is not possible.
𝟑

Reason : Hypotenuse is the largest side in any right angled triangle.


(a) Both assertion (A) and reason (R) are true and reason (R) is the correct explanation of
assertion (A).
(b) Both assertion (A) and reason (R) are true but reason (R) is not the correct explanation of
assertion (A).
(c) Assertion (A) is true but reason (R) is false.
(d) Assertion (A) is false but reason (R) is true.

SECTION B
Q21. Sides AB and BC and median AD of ∆ABC are respectively proportional to
sides PQ and QR and median PM of ∆PQR. Show that ∆ABC ~ ∆PQR

Q22. In figure 6, find the area of the shaded region, enclosed between two concentric circles of
𝟐𝟐
radii 7 cm and 14 cm where AOC  40 0 . (use π= 𝟕
)
OR

In Fig.2, a quadrilateral ABCD is drawn to circumscribe a circle, with centre O, in such a way
that the sides AB, BC, CD and DA touch the circle at the points P, Q, R and S respectively.

Prove that AB+CD= BC+DA.

Q23. Find the value of k, so that the pair of linear equation has unique solution:
. 𝟐𝐱 + 𝐤𝐲 = 𝟏𝟏 𝐚𝐧𝐝 𝐱 − 𝟐𝐲 = −𝟏𝟐
Q24. Prove that length of the tangents drawn from an external points to a circle are equal.

Q25. If 3 cot A = 4, check whether (1-tan2 A)/(1+tan2 A) = cos2 A – sin 2 A or not.

OR
Evaluate :
(i) sin 60° cos 30° + sin 30° cos 60°

SECTION C

Q26. Show that√𝟓is irrational.


Q27. Find the zeroes of 𝟔𝐱 𝟐 – 𝟑 – 𝟕𝐱, and verify the relationship between the zeroes and the
coefficients..

1  sin x
Q28. Prove that  sec x  tan x
1  sin x
or
cos A sin A
+ = sin A + cos A
1  tan A 1  cot A

Q29.One card is drawn from a well-shuffled deck of 52 cards. Find the probability of getting

(i) a king of red colour


(ii) a face card
(iii) a red face card

Q30.Prove that the parallelogram circumscribing a circle is a rhombus

Q31.Solve the pair of linear equations


3x-5y-4 =0
9x =2y +7

OR

Draw the graphs of the equations x – y + 1 = 0 and 3x + 2y – 12 = 0. Determine the coordinates of the
vertices of the triangle formed by these lines and the x-axis, and shade the triangular region.

SECTION D

Q32. State and prove BASIC PROPORTIONALITY THEOREM.

Q33. Find Median of the following data

Marks 0 – 10 10 - 20 20 - 30 30 - 40 40 - 50 50 – 60

Number of students 5 4 8 10 15 18

OR
If the mean of the following distribution is 50 find the value of p.

Class 0 - 20 20 - 40 40 - 60 60 - 80 80 - 100

Frequency 17 28 32 p 19
34. A toy is in the form of a cone of radius 3.5 cm mounted on a hemisphere of same radius. The
total height of the toy is 15.5 cm. Find the total surface area of the toy.

35.The altitude of a right triangle is 7 cm less than its base. If the hypotenuse is 13 cm, find the
other two sides.

OR
In a class test, the sum of Shefali’s marks in Mathematics and English is 30. Had she got 2
marksmore in Mathematics and 3 marks less in English, the product of their marks would have
been 210. Find her marks in the two subjects.

SECTION-E

36.Salary : In investigating different job opportunities, you find that firm A will start you at Rs
25,000 per year and guarantee you a raise of Rs 1,200 each year whereas firm B will start
you at Rs 28,000 per year but will guarantee you a raise of only Rs 800 each year.
(i) Over a period of 15 years, how much would you receive from firm A?

(a) 501000 (b )504000 (c) 35800 (d) 35200

(ii) Over a period of 15 years, how much would you receive from firm B?

(a) 501000 (b )504000 (c) 35800 (d) 35200

(iii) What would be your annual salary at firm A for the tenth year?

(a) 501000 (b )504000 (c) 35800 (d) 35200


OR
(iii) What would be your annual salary at firm B for the tenth
(a) 501000 (b )504000 (c) 35800 (d) 35200

37.Rohan wants to measure the distance of a pond during the visit to his native.
He marks points A and B on the opposite edges of a pond as shown in the
figure below. To find the distance between the points, he makes a right-angled
triangle using rope connecting B with another point C are a distance of 12m,
connecting C to point D at a distance of 40m from point C and the connecting D
to the point A which is are a distance of 30m from D such the ˂ ADC=90°
1. Which property of geometry will be used to find the distance AC?

a) Similarity of triangles b) Thales Theorem

c) Pythagoras Theorem d) Area of similar triangles

2. Which is the following does not form a Pythagoras triplet?

a) (7,24,25) b) (15,8,17) c) (5,12,13) d) (21,20,28)

3. Find the length AB?

a) 12m b) 38m c) 50m d) 100m

OR

3. Find the length of the rope used.

a) 120m b) 70m c) 82m d) 22m

38. Ajay, Bhigu and Colin are fast friend since childhood. They always want to sit in a row in the
classroom . But teacher doesn’t allow them and rotate the seats row-wise everyday. Bhigu is very
good in maths and he does distance calculation everyday. He consider the centre of class as origin
and marks their position on a paper in a co-ordinate system. One day Bhigu make the following
diagram of their seating position.
(i) What are the coordinates of point A?
a) ( -2,2 )b) ( 2,4 ) c) ( 4,2 ) d) ( 3,2 )

(ii) What is the distance of point A from origin ?


a) 2√𝟐 b) 3√𝟐c) 4√𝟐 d) 5√𝟐

(iii) What is the distance between B and C ?


a) 3√𝟓 b) 2√𝟓 c) 4√𝟓 d) 5√𝟓

OR
(iii) A point D lies on the line segment between points A and B such that AD : : DB
= 4 3 . What are the coordinates of point D
𝟏𝟎 𝟐
a) (-4,5) b) (-3,6) c) (- 𝟕 ,-𝟕)d) (-3,7)
KENDRIYA VIDYALYA SANGATHAN, JAMMU REGION
Sample Paper Set - 10

CLASS – X SUBJECT -Mathematics Basic (241)


Time : 3 Hours MM: 80
General Instructions:
1. This Question Paper has 5 Sections A-E.
2. Section A has 20 MCQs carrying 1 mark each
3.Section B has 5 questions carrying 02 marks each.
4. Section C has 6 questions carrying 03 marks each.
5. Section D has 4 questions carrying 05 marks each.
6. Section E has 3 case based integrated units of assessment (04 marks each) with
subparts of the values of 1, 1 and 2 marks each respectively.
7. All Questions are compulsory. However, an internal choice in 2 Qs of 5 marks, 2 Qs
of 3 marks and 2 Questions of 2 marks has been provided. An internal choice has been
provided in the 2marks questions of Section E
8. Draw neat figures wherever required. Take π =22/7 wherever required if not stated.
SECTION A
SECTION A consists of 20 questions of 1 mark each.

1 If one zero of a quadratic polynomial kx2+3x+k is 2, then the value of k is 1


(a) 5/6 (b)-5/6 (c) 6/5 (d)-
6/5

2 The curved surface of the cylinder is 88 cm2. The height of the cylinder is 5 m, what 1
is the radius of the cylinder?
a) 6 cm b) 4 cm c) 1 cm (d)2. 8cm

3 A cylinder and a cone are of same base radius and of same height. The ratio of the 1
volume of the cylinder to that of the cone is
(a) 2 : 1 b) 3 : 1 (c) 2 : 3 (d)1:3

4 C is the mid point of PQ, if Pis (4,x),C is (y,-1) and Q is (-2,4), then x and y 1
respectively are
(a) -6 and 1 (b)-6 and 2 (c) 6 and -1 (d) 6
and -2

5 Choose the correct one 1


(a) 3median = 2 mode + mean (b) 2mean= 3median +mode (c)
2mean=3median-mode (d) none of the above

6 The possibility to come out 7 on dice is 1


(a) sure event (b) impossible event (c) possible event
(d) complementary event

7 The maximum value of Sin is 1


(a)1/2 (b)√3 /2 (c)1
(d) 1/√2
8 If LCM=120,HCF=4 then the product of two numbers is 1
(a) 30 (b) 480 (c) 240 (d)60

9 7× 11× 13 + 13 is......... 1
(a) A prime number (b) a composite number (c ) an odd number (d) divisible
by 5
1 Volume of two cubes is in the ratio of 8 : 125. The ratio of their surface areas is 1
0
a) 8:125 b) 2:5 c) 4:2 d) 16:25
1 If two dice are thrown together, what is the probability of getting an even number on 1
1 one dice and an odd number on the other dice?
(a)1/4 (b)3/5 (c)3/4 (d)1/2

1 1
2 The 4th term from the end of an AP -11,-8,-5,----------------,49 is
(a)37 (b) 40 (c)43 (d) 58

1 Which of the following is a rational number? 1


3 (a) √2 (b) ∏ (c) 1/3 (d) 0.12012001200012000...
1 From an external point P, tangents PA and PB are drawn to a circle with centre O. If 1
4 CD is the tangent to the circle at a point E and PA=14cm.The perimeter of ΔPCD is
a)14 (b)21 (c)28 (d) 35

1 QP is a tangent to a circle with centre O at a point P on the circle If ΔOPQ is isosceles 1


5 then ˂OQR=
(a)30 (b) 45 (c)60 (d)
90
,m
1 sin 2B = 2 sin B is true when B is equal to 1
6 (a) 90° (b) 60° (c) 30° (d) 0°

1 If the point P(6,2) divides the line segment joining A(6,5) and B(4,y) in the ratio of 1
7 3:1 then the value of y is
(a)4 (b) 3 (c)2 (d 1

1 The pair satisfying 2x + y = 6 is 1


8 (a) (3,4) (b) (3,-3) (c) (2,1) (d) (2,2)

Direction for questions 19 & 20 a statement of Assertion (A) is followed by a statement


of Reason (R) .Choose the correct option.
1 Assertion(A): In an equilateral triangle of side 3√3 cm, then the length of the altitude 1
9 is 4.5cm.

Reason(R):If a ladder 10 cm long reaches a window 8 m above the ground, then the
distance of the foot of ladder from the base of the wall is 6 cm

(a) Both assertion (A) and reason (R) are true and reason (R) is the correct
explanation of assertion (A).
(b) Both assertion (A) and reason (R) are true but reason (R) is not the correct
explanation of assertion (A).
(C) Assertion (A) is true but reason (R) is false.
(d) Assertion (A) is false but reason (R) is true.

2 Assertion: The value of sin600 cos300 + sin300 cos600 is 1 1


0 Reason: sin900=1 and cos900=0

(a) Both assertion (A) and reason (R) are true and reason (R) is the correct
explanation of assertion (A).
(b) Both assertion (A) and reason (R) are true but reason (R) is not the correct
explanation of assertion (A).
(C) Assertion (A) is true but reason (R) is false.
(d) Assertion (A) is false but reason (R) is true.

SECTION B
SECTION B consists of 5 questions of 2 marks each

2 Prove that in two concentric circles, the chord of the larger circle, which touches the 2
1 smaller circle, is bisected at the point of contact.
Or
Prove that the tangents drawn at the ends of a diameter of a circle are parallel
2 The angle of elevation of the top of a building from the foot of a tower is 30°, If the 2
2 tower is 30m high. find distance between tower and building

2 Which term of the A.P. 3, 8, 13, 18, is 78? 2


3 Or
Find the number of terms in each of the AP 7, 13, 19, …, 205
2 2 cubes each of volume 64 cm3 are joined end to end. Find the surface area of the 2
4 resulting cuboid.
2 Find the area of a quadrant of a circle whose circumference is 22 cm. 2
5
Section C
SECTION C consists of 6 questions of 3 marks each

2 Solve by elimination method. find a and b. 3


6 4a+6b=24
6a+4b=36

2 If tan (A + B) = √3 and tan (A – B) = 1/√3 ,0° < A + B ≤ 90°; A > B, find A and B. 3
7
2 A survey conducted on 20 houses in an area by a group of people resulted in the
8 subsequent frequency table for the number of family members in a house:
Size of family 1-3 3-5 5-7 7-9 9-11
No. of families 7 8 2 2 1 3
Find mode of this data

2 Find roots by quadratic formula 3


9 √2x2+5x+3√2.

3 From a point T outside a circle of centre O, tangents TP and TQ are drawn to circle. 3
0 Prove that OT is the right bisector of line segment PQ.

3 Find the LCM and HCF of the following pairs of integers 26 and 91 and verify that 3
1 LCM × HCF = product of the two numbers.
Or
Prove that 3+√5 are irrational.
Section:-D
SECTION D consists of 4 questions of 5 marks each

3 Statement and prove Thales theorem 5


2
3 The altitude of a right triangle is 7 cm less than its base. If the hypotenuse is 13 cm, 5
3 find the other two sides.
Or
Sum of the areas of two squares is 468m2b. If the difference of their perimeters is 24 m,
find the sides of the two squares.

3 Rain water which falls on a flat rectangular surface of length 6 m and breadth 4 m is 5
4 transferred into a cylindrical vessel of internal radius 20 cm. What will be the height
of water in the cylindrical vessel if the rain fall is 1 cm. Give your answer to the
nearest integer. (Take π = 3.14)
3 From a deck of ship of cards, all queens are removed. a card is selected randomly 5
5 from the remaining cards, what is the probability that the selected card is a
(a)Red jack (b) black queen
(c) an ace (d) a spade of king (e) a face
card

Section:-E
SECTION D consists of 4 questions of 5 marks each

3 A Satellite flying at height h is watching the top o the two tallest mountains in
6 Uttarakhand and Karnataka, them being Nanda Devi(height 7,816m) and
Mullayanagiri (height 1,930 m). The angles of depression from the satellite, to the tp
of Nanda Devi and Mullayanagiri are 30° and 60° respectively. If the distance
between te peaks of the two mountains is 1937 km, and the satellite is vertically
above the midpoint of the distance between the two mountains.
1
(i) The distance of the satellite from the top of Nanda Devi is.......... 1
(ii) The distance of the satellite from the top of Mullayanagiri is..... 2
(iii) The distance of the satellite from the ground is........
Or
(iv) What is the angle of elevation if a man is standing at a distance of 7816m 2
from Nanda Devi?

3 The top of a table is shown in the figure given below:


7

1
(i) The coordinates of the points H and G are respectively

(ii) The distance between H and G 2

(iii) The distance between the points A and B is


Or

The coordinates of the midpoint of line segment joining points M and Q are
3
Your friend Veer wants to participate in a 200m race. He can currently run that
8
distance in 51 seconds and with each day of practice it takes him 2 seconds less. He
wants to do in 31 seconds.

1
(i)Frame the AP for the given situation
2
(ii)What is the minimum number of days he needs to practice till his goal is achieved

(ii) Check whether 30 is term of the AP of the above given situation 2

Or
(iii) If nth term of an AP is given by an = 2n + 3 then common difference of an AP is
KENDRIYA VIDYALYA SANGATHAN, JAMMU REGION
Sample Paper Set - 11

CLASS – X SUBJECT -Mathematics Basic (241)


Time : 3 Hours MM: 80

General Instructions:
1. This Question Paper has 5 Sections A-E.
2. Section A has 20 MCQs carrying 1 mark each
3. Section B has 5 questions carrying 02 marks each.
4. Section C has 6 questions carrying 03 marks each.
5. Section D has 4 questions carrying 05 marks each.
6. Section E has 3 case based integrated units of assessment (04 marks each) with
subparts of the values of 1, 1 and 2 marks each respectively.
7. All Questions are compulsory. However, an internal choice in 2 Qs of 5 marks, 2 Qs
of 3 marks and 2 Questions of 2 marks has been provided. An internal choice has been
provided in the 2marks questions of Section E
8. Draw neat figures wherever required. Take π =22/7 wherever required if not stated.

SECTION A
Section A consist of 20 questions of 1 mark each.
S.N Mar
o. ks
LCM of the given number ‘x’ and ‘y’ where y is a multiple of ‘x’ is given by 1
1 (a) x
(b) y
(c) xy
(d) x/y
If the radius of base of a right circular cylinder is halved, keeping the height 1
2 same, the ratio of the volume of the reduced cylinder to that of the original
cylinder is:
(a) 2 : 3
(b) 3 : 4
(c) 1 : 4
(d) 4 : 1
The HCF and LCM OF 12,21,15 respectively are 1
3 a) 3,140 b) 3,420 c) 12,420 d) 420,3
4 If the quadratic polynomial has two roots as 1 and -1, then the polynomial will 1
be
a) x²+2x+1
b) x²+1-2x
c) x²+1
d) x²-1
5 2 tan 30°/(1 + tan230°) = 1
a) sin 60° b) cos 60° c) tan 60° d) sin 30°

6 A cylindrical pencil sharpened at one edge is the combination of 1


(a) two cylinders
(b) a hemisphere and a cylinder
(c) a cone and a cylinder
(d) frustum of a cone and a cylinder
7 Which one is rational? 1
(a)√3-√2. b) √12/√3. c) √3. d) none
8 When a die is thrown, the probability of getting an odd number less than 3 is 1
(a)1/6 (b) 1/3 c)½ d) 0
9 The mean and mode of a data is 24 and 12 respectively then the median is 1
a)22. b) 20. c) 18. d) 25
10 If the distance between the points A(2, -2) and B(-1, x) is equal to 5, then the 1
value of x is:
(a) 2 b) -2 c) 1 d) -1
11 If triangles ABC and DEF are similar and AB=4 cm, DE=6 cm, EF=9 cm and 1
FD=12 cm, the perimeter of triangle ABC is:
(a) 22 cm b) 20 cm c) 21 cm d) 18 cm
12 In the given figure, AB and AC are tangents to the circle with centre O such 1
that ∠BAC = 40°, then ∠BOC is equal to [AI2011]

(a) 40°
(b) 50°
(c) 140°
(d) 150
13 5 tan² A – 5 sec² A + 1 is equal to 1
(a) 6 b) -5 c) 1 d) -4
14 In the figure PA and PB are tangents to the circle with centre O. If ∠APB = 1
60°, then ∠OAB is

(a) 30° (b) 60° (c) 90° (d) 15


15 If O(p/3, 4) is the midpoint of the line segment joining the points P(-6, 5) and 1
Q(-2, 3), the the value of p is:
(a) 7/2
(b) -12
(c) 4
(d) -4
16 In ∆ABC, AB = 6√3 cm, AC = 12 cm and BC = 6 cm. The angle B is 1
(a) 120°
(b) 60°
(c) 90°
(d) 45°
17 The probability of winning a game is 0.06. what is the probability of losing the 1
game?
a) 0.05 b)0.04 c)0.1 d) 0.94
18 It is given that ∆ ABC ≅ ∆ FDE and AB = 5 cm, ∠B = 40° and ∠A = 80°. 1
Then which of the following is true?
(a) DF = 5 cm, ∠F = 60°
(b) DF = 5 cm, ∠E = 60°
(c) DE = 5 cm, ∠E = 60°
(d) DE = 5 cm, ∠D = 40
Direction for questions 19 & 20 a statement of Assertion (A) is followed by a statement
of Reason (R) .Choose the correct option.
19 Assertion (A): The fourth term from end of the AP -11, -8 , -5,......49 is 40. 1
Reason(R): If the nth term of an AP -1, 4,9,14... is 129, then the value of n is
100.
(a) Both assertion (A) and reason (R) are true and reason (R) is the correct
explanation of assertion (A).
(b) Both assertion (A) and reason (R) are true but reason (R) is not the correct
explanation of assertion (A).
(c) Assertion (A) is true but reasons (R) is false.
(d) Assertion (A) is false but reasons (R) is true.
20 Assertion (A):- If circumference of two circles is equal, then their areas will be 1
equal.
Reason(R): if the areas of two circles are equal, then their circumference is
equal.
(a) Both assertion (A) and reason (R) are true and reason (R) is the correct
explanation of assertion (A).
(b) Both assertion (A) and reason (R) are true but reason (R) is not the correct
explanation of assertion (A).
(c) Assertion (A) is true but reasons (R) is false.
(d) Assertion (A) is false but reasons (R) is true.
Section B
A Section B consists of 5 questions of 2 marks each. 2
21 A solid sphere of radius 3cm is melted and then cast into small balls each of 2
radius 0.3cm. Find the no. of balls thus obtained.
22 The angle of elevation of the top of a building from a point on the ground, 2
which is 30 m away from the foot of the building, is 30°. Find the height of
building.

23 AOB is a diameter of a circle with centre O and AC is a tangent to the circle at 2


A. If <AOB=130⁰, then find <ACO
OR
From a point Q, the length of the tangent to a circle is 24 cm and the distance of
Q from the centre is 25 cm. Find the radius of the circle
24 In a two digit number, the digit in the unit place is twice of the digit in the tenth 2
place. If the digits are reversed, the new number is 27 more than the given
number. Find the number.
OR
If 1 is a root of the equations ay2 + ay + 3 = 0 and y2 + y + b = 0, then find the
value of ab.
25 A car has two wipers which do not overlap. Each wiper has a blade of length 25 2
cm sweeping through an angle of 115⁰. Find the total area cleaned at each
sweep of the blades.
SECTION C
Section C consists of 6 questions of 3 marks each.
26 Prove the identity where the angles involved are acute angles for which the 3
expressions are defined.
(CosecA- CotA )²= (1-cosA)/(1+cosA)
OR
A 7 m long flagstaff is fixed on the top of a tower standing on the horizontal
plane. From a point on the ground, the angles of elevation of the top and
bottom of the flagstaff are 60° and 45° respectively. Find the height of the
tower correct to one place of decima
27 Two different dice are thrown together. Find the probability of 3

I) getting a number greater than 3 on each die.

2)getting a total of 6 or 7 of the numbers on two dice


28 Prove that the parallelogram circumscribing a circle is a rhombus. 3

29 Solve for x and y: 3


x+2y-3=0
3x-2y+7=0
OR
Find the value of a and b if the following pair of linear equations has infinite
number of solutions.
3x-(a+1)y= 2b -1 ;
5x+(1-2a)y= 3b
30 Find the zeroes of the quadratic polynomial 6x²-3-7x and verify the relationship 3
between zeroes and coefficients of the polynomial.
31 Prove √5 is irrational. 3
SECTION D
Section D consists of 4 questions of 5 marks each.
32 State and prove Basic proportionality theorem. 5
33 A thief runs with a uniform speed of 100m/min. After one minute,a police runs 5
after the thief to catch him. He goes with a speed of 100m/min in the first
minute and increases his speed by 10 m/min every succeeding minute. After
how many minutes the policeman will catch the thief?
OR
A passenger, while boarding the plane, slipped from the stairs and got hurt. The
pilot took the passenger in the emergency clinic at the airport for treatment.
Due to this, the plane got delayed by half an hour. To reach the destination
1500km away in time, so that the passengers could catch the connecting flight,
the speed of the plane was increased by 250km/hour than the usual speed. Find
the usual speed of the plane.
34 A card is drawn at random from a well- shuffled deck of playing cards. Find the 5
probability that the card drawn is
i) A card of spade
ii) A card of king of red colour
iii) A black ace
iv) A face card
v) Not a face card
OR
The mean of the following frequency table is 57.6. But the frequencies f1 and f2 are
missing. Find the value of missing frequencies f1 and f2.
Class Interval 0−20 20−40 40−60 60−80 80−100 100−120 Total
Frequency 7 f1 12 f2 8 5 50
35 A tent is in the shape of a cylinder surmounted by a conical top. If the height 5
and diameter of the cylindrical part are 2.1 m and 4 m respectively, and the
slant height of the top is 2.8 m, find the area of the canvas used for making the
tent. Also, find the cost of the canvas of the tent at the rate of Rs 500 per m2
SECTION E
36 Rehan lives in Meghalaya. Satellite image of his colony is shown in given
figure . In this view, his house is pointed out by a hage, which is situa6at the
point of intersection of x and y-axis. If he goes 2cm east and 3 cm north from
the house ,then he reaches a grocery store, if he goes 4cm west and 6 cm south
from the house, then he reaches to his office. If he goes 6cm east and 8cm south
from the house then he reaches to a food court. If he goes 6cm west and 8cm

1
2

north from the house , he reaches to his kid’s school.

1. Find the distance between grocery store and food court?(1mark)


2 .If the grocery store and office lie on a line, what is the ratio of distance of
house from grocery store to that from office? (2 marks)
OR
What shape is formed by the coordinates of positions of school l, grocery store,
food court and office?
3 .Find the ratio of distance of house from school to food court? 1( mark)

37
India is competitive manufacturing location due to the low cost of manpower
and strong technical and engineering capabilities contributing to higher quality
production runs. The production of TV sets in a factory increases uniformly by
a fixed number every year. It produced 16000 sets in 6th year and 22600 in 9th
year.

Based on the above information, answer the following questions: 1

1. Find the production during first year.(1Mark)


1
2. Find the production during 8th year.(1 Mark)
2

3. Find the production during first 3 years.(2 Marks)


OR
2

In which year, the production is Rs 29,200.

38 An observer on the top of a 30 m tall light house observes a ship at an angle of


depression 30⁰ coming towards the base of light house along a straight line
joining the ship and the base of the light house. The angle of ship changes to
45⁰ after 10 seconds

1
1. The distance of the ship from the base of the light house when angle of
1
depression is 30⁰ is?
2. The distance of the ship from the base of the light house after 10 2
seconds from the initial position when the angle of depression changes
to 45⁰?
3. The distance between the two positions of ship after 10 seconds is?
KENDRIYA VIDYALYA SANGATHAN, JAMMU REGION
Sample Paper Set - 12

CLASS – X SUBJECT -Mathematics Basic (241)


Time : 3 Hours MM: 80

General Instructions:
1. This Question Paper has 5 Sections A, B, C, D, and E.
2. Section A has 20 Multiple Choice Questions (MCQs) carrying 1 mark each.
3. Section B has 5 Short Answer-I (SA-I) type questions carrying 2 marks each.
4. Section C has 6 Short Answer-II (SA-II) type questions carrying 3 marks each.
5. Section D has 4 Long Answer (LA) type questions carrying 5 marks each.
6. Section E has 3 Case Based integrated units of assessment (4 marks each) with sub-parts of
the values of 1, 1 and 2 marks each respectively.
7. All Questions are compulsory. However, an internal choice in 2 Qs of 2 marks, 2 Qs of 3
marks and 2 Questions of 5 marks has been provided. An internal choice has been provided in
the 2 marks questions of Section E.
8. Draw neat figures wherever required. Take π =22/7 wherever required if not stated.

SECTION – A

Q.No. Marks

1. The LCM of 22×33 and 23×32 is 1


(a)33 (b) 23 (c)23×33 (d) 22×32

2. The HCF of two numbers is 18 and their product is 12960. Their LCM will be 1
(a) 420 (b) 600 (c) 720 (d) 800
3. The prime factorisation of 3825 is 1
(a) 3×52 ×21 (b) 3 2 ×52 ×35 (c) 3 2 ×52 ×17 (d) 3 2 ×25×17
4. Graphically, the pair of equations given by 1
4x – 2y + 18 = 0
2x – y + 9 = 0
represents two lines which are
(a) intersecting at exactly one point. (b) parallel.
(c) coincident. (d) intersecting at exactly two
points.
5. Find the values of k for which the quadratic equation 3x2 + kx + 3 = 0 has real and 1
equal roots.
(a) ±6 (b)+ √24 (c) -√24 (d) +6
6. In an A.P. if d= -4, n= 7, an = 4, then a is 1
(a) 7 (b) 20 (c) 6 (d) 28

7. The distance of the point P(2,3) from X- axis is 1


(a) 2 (b) 3 (c) 1 (d) 5

8. 1
In ΔABC, if DE || BC, AD = x, DB = x – 2, AE = x + 2 and EC = x – 1, then value of
x is
(a) 3 (b) 4 (c) 5 (d) 3.5
9. In the given figure, ΔACB ~ ΔAPQ. If AB = 6 cm, BC = 8 cm, and PQ = 4 cm then 1
AQ is equal to

(a) 2 cm
(b) 2.5 cm
(c) 3 cm
(d) 3.5 cm
10. If Fig., AB is a chord of the circle and AOC is its diameter such that ∠ACB = 50°. If 1
AT is the tangent to the circle at the point A, the ∠BAT is equal to

(a) 65°
(b) 60°
(c) 50°
(d) 40°
11. Find the angle of elevation of the sun when the shadow of a pole h m high is √3 h m
long.
(a) 45◦
(b) 30◦
(c) 60◦
(d) 90◦
12. If sinƟ = x and secƟ = y , then tanƟ is 1
(a) xy
(b) x/y
(c) y/x
(d) 1/xy
13. 2 tan 30°/(1 + tan230°) = 1
(a) sin 60° (b) cos 60° (c) tan 60° (d) sin 30°

14. The surface area of a sphere is 616 cm2. Its radius is 1


(a) 7 cm
(b) 14 cm
(c) 21 cm
(d) 28 cm
15. A surahi is the combination of 1
(a) a sphere and a cylinder
(b) a hemisphere and a cylinder
(c) two hemispheres
(d) a cylinder and a cone
16. If the difference between the circumference and the radius of a circle is 37cm 1
,∏=22/7, the circumference (in cm) of the circle is
(a) 154
(b) 44
(c) 14
(d) 7
17. The class mark of the class 25-35 is 1
(a) 30
(b) 29.5
(c) 30.5
(d) 29
18. If P (E) denotes the probability of an event E, then 1
(a) 0< P(E) ⩽1
(b) 0 < P(E) < 1
(c) 0 ≤ P(E) ≤1
(d) 0 ⩽P(E) <1
Direction for questions 19 & 20 a statement of Assertion (A) is followed by a statement of Reason (R)
.Choose the correct option.
19. Assertion(A) : The coordinates of mid-points of P(5,8) and Q(-7,-3) is (-1,2.5) 1
𝑥2−𝑥1 𝑦2−𝑦1
Reason(R) : Coordinates of mid point A(x1,y1) and B(x2,y2) are (
2
, )
2
(a) Both Assertion(A) and Reason (R ) are true and Reason is the correct explanation
of Assertion.
(b) Both Assertion(A) and Reason (R ) are true and Reason is not the correct
explanation of Assertion.
(c) Assertion (A) is true but Reason (R ) is false.
(d) Assertion (A) is false but Reason (R ) is true.
20. Assertion (A): If P(E) =0.30, then the probability of ‘not E’ is 0.70. 1
Reason (R) : If two dice are thrown togrther, then the probability of getting a doublet is
5/6.
(a) Both assertion (A) and reason (R) are true and reason (R) is the correct explanation
of assertion (A).
(b) Both assertion (A) and reason (R) are true but reason (R) is not the correct
explanation of assertion (A).
(c) Assertion (A) is true but reasons (R) is false.
(d) Assertion (A) is false but reasons (R) is true.
SECTION –B

21. For what value of p does the pair of equation given below has no solution. 4x + py + 8 2
= 0 and 2x +2y + 2 = 0
22. In the figure, if LM || CB and LN || CD, prove that AM/AB = AN/AD 2

OR
In the figure, if ΔABE ≅ ΔACD, show that ΔADE ~ ΔABC.
23. PQ and RS are the tangents drawn at the end points of the diameter AB of the circle with 2
centre O. Prove that PQ RS

.
24. A ladder leaning against wall , makes an angle of 30◦with the horizontal. If the foot of 2
the ladder is 5m away from the wall, find the length of the ladder.

25. A circle of radius 21cm is divided into 10 equal sectors by using 5 diameters. Find area 2
of each sector.
OR
The wheels of a car are of diameter 80 cm each. How many complete revolutions does
each wheel make in 10 minutes when the car is travelling at a speed of 66 km per hour?
SECTION-C

26. Prove that √7 is an irrational number. 3

27 Find the zeroes of the following quadratic polynomial and verify the relationship 3
between the zeroes and their coefficients: 4u2 + 8u
28. Find the nature of the roots of the quadratic equation: 3x 2 -5x+2. If the real roots exist, 3
find them by using quadratic formula.

OR
The diagonal of a rectangular field is 60 metres more than the shorter side. If the
longer side is 30 metres more than the shorter side, find the sides of the field.

29. If ∠A and ∠B are acute angles such that cos A = cos B, then show that ∠ A = ∠ B. 3

30. Prove that opposite sides of a quadrilateral circumscribing a circle subtend 3


supplementary angles at the centre of the circle.

OR
A quadrilateral ABCD is drawn to circumscribe a circle (see figure). Prove that AB +
CD = AD + BC.

31 To find out the concentration of SO2 in the air (in parts per million, i.e., ppm), the data
was collected for 30 localities in a certain city presented below:
3
Find the mean concentration of SO2 in the air.

Concentration 0.00 – 0.04- 0.08 – 0.12 – 0.16– 0.20 –


of SO2 ( in 0.04 0.08 0.12 0.16 0.20 0.24
ppm)
Frequency 4 9 9 2 4 4

SECTION-D
32. Sides AB and BC and median AD of a triangle ABC are respectively proportional to 5
sides PQ and QR and median PM of ΔPQR. Show that ΔABC ~ ΔPQR.
OR
State and prove Basic Proportionality Theorem ,and use it to find x in the figure
.
33. A pen stand made of wood is in the shape of a cuboid 5
with four conical depressions to hold pens. The
dimensions of the cuboid are 15 cm by 10 cm by 3.5
cm. The radius of each of the depressions is 0.5 cm
and the depth is 1.4 cm. Find the volume of wood in the
entire stand.

OR
A tent is in the shape of a cylinder surmounted by a conical top. If the height and
diameter of the cylindrical part are 2.1 m and 4 m respectively, and the slant height of
the top is 2.8 m, find the area of the canvas used for making the tent. Also, find the cost
of the canvas of the tent at the rate of Rs 500 per m2.
34. Places A and B are 100 km apart on a highway. One car starts from A and another from 5
B at the same time. If the cars travel in the same direction at different speeds, they meet
in 5 hours. If they travel towards each other, they meet in 1 hour. What are the speeds of
the two cars?

One card is drawn from a well-shuffled deck of 52 cards. Find the probability of getting 5
35. (i) a jack of black colour
(ii) a face card
(iii) a red face card
(iv) the king of clubs
(v) a card with number 9
SECTION-E
CASE STUDY QUESTIONS

36. HCV Pvt, Ltd is a leading LED TV manufacturing company in India. Its production of
LED TV in India increased uniformly by fixed number every year. It produced 16000
sets in 6th year and 22600 in 9th year.

(a) Find the production in the 1st year. 1


(b) Find the production during first 2 years.
(c) Find the difference of the production during 7th year and 4th year. 1
OR 2
In which year, the production is 29,200.
37.

A theodolite is a precision optical instrument for measuring angles between


Designated visible points in the horizontal and vertical planes. The traditional
use has been for land surveying, but they are also used extensively for building and
infrastructure construction, and some specialized applications such
as meteorology and rocket launching. It consists of a
moveable telescope mounted so it can rotate around horizontal and
Vertical axes and provide angular readouts.
A student is using a theodolite, situated at some height from the ground at point P and
75 m away from a building, as shown in figure.
1

2
Observe the figure and answer the following:
(a)If α1=60◦, find the distance between top of the building and theodolite?
(b)If α2 =30◦,find the height of theodolite from the ground?
c) Find height of the building .
OR
With respect to angle α1, find the perpendicular distance in the figure.

38

To conduct sports day activities, in your rectangular shaped school ground ABCD, lines
have been drawn at a distance of 1 m each. 100 flower pots have been placed at a
distance of 1 m from each other along AD, as shown in the above figure. Niharika runs
1/4th the distance AD on the 2nd line and fix a Green flag, Preet runs 1/5th the distance
AD on the eighth line and fix a Red flag.
(a)What is the point of GREEN flag?
(b)What is the point of RED flag? 1
(c )What is the distance between two flags? 1
OR 2
What is the Midpoint of the line segment joining both the flags?
KENDRIYA VIDYALYA SANGATHAN, JAMMU REGION
Sample Paper Set - 13

CLASS – X SUBJECT -Mathematics Basic (241)


Time : 3 Hours MM: 80

General Instructions:
1. This Question Paper has 5 Sections A, B, C, D, and E.
2. Section A has 20 Multiple Choice Questions (MCQs) carrying 1 mark each.
3. Section B has 5 Short Answer-I (SA-I) type questions carrying 2 marks each.
4. Section C has 6 Short Answer-II (SA-II) type questions carrying 3 marks each.
5. Section D has 4 Long Answer (LA) type questions carrying 5 marks each.
6. Section E has 3 Case Based integrated units of assessment (4 marks each) with sub-parts of the values of 1, 1
and 2 marks each respectively.
7. All Questions are compulsory. However, an internal choice in 2 Qs of 2 marks, 2 Qs of 3 marks and 2
Questions of 5 marks has been provided. An internal choice has been provided in the 2 marks questions of
Section E.

Section A 1 x 20 = 20
1. HCF of 865 and 255 is 1

(a) 15 (b) 25 (c) 5 (d) 10


2. If two positive integers a and b are written as a = x3y2 and b = xy3; x, y are prime numbers, 1
then HCF (a, b) is
(a) xy (b) xy2 (c) x3y3 (d) x2y2

3. It is given that the difference between the zeros of 4x2 – 8kx + 9 is 4 and k>0, 1
then k = ?
(a) 1/2 (b) 3/2 (c) 5/2 (d) 7/2
4. 1
The pair of equations x + 2y + 5 = 0 and –3x – 6y + 1 = 0 have
(a) a unique solution (b) exactly two solutions
(c) infinitely many solutions (d) no solution

5. The quadratic equation 2x2 – 5x + 1 = 0 has 1


(a) two distinct real roots (b) two equal real roots
(c) no real roots (d) more than 2 real roots

6. The distance of the point P (–6, 8) from the origin is 1


(a) 8 (b) 2√7 (c) 10 (d) 6

7. If the distance between the points (2, –2) and (–1, x) is 5, one of the values 1
of x is
(a) –2 (b) 2 (c) –1 (d) 1
8. 1 – cos2A is equal to: 1
(a) sin2A (b) tan2A (c) 1 – sin2A (d) sec2A

Page 1 of 6
9. The value of sin60°.cos30° + sin30°.cos60° is: 1
(a) 0 (b) 1 (c) 2 (d) 4
10. If cos X = 2/3 , then tan X is equal to: 1
(a) 0 (b) √2/√3 (c) √5/2. (d) √5/3
11. D and E are respectively the points on the sides AB and AC of a triangle ABC such that AD = 1
2 cm, BD = 3 cm, BC = 7.5 cm and DE ∥ BC. Then, length of DE (in cm) is

(a) 2.5 (b) 3 (c) 5 (d) 6


o
12. If angle between two radii of a circle is 130 , the angle between the tangents at the ends of the 1
radii is :

(a) 90º (b) 50º (c) 70º (d) 40º

13. The area of a circle is 49 𝜋 cm2. Its circumference is 1

(a) 7 𝜋 cm (b) 14 𝜋 cm (c) 21 𝜋 cm (d) 28 𝜋 cm

14. The perimeter of a circular field is 242 m. The area of the field is 1
(a) 9317 m2 (b) 18634 m2 (c) 4658.5 m2 (d) none of these

15. Volumes of two spheres are in the ratio 64:27. The ratio of their surface areas is 1
(a) 3 : 4 (b) 4 : 3 (c) 9 : 16 (d) 16 : 9

16. In the following distribution:- 1


Marks Less than Less than Less than Less than Less than Less than
20 40 60 80 100 120
No. of 4 12 25 56 74 80
student
the modal class is
(a) 20 – 40 (b) 40 – 60 (c) 60 – 80 (d) 80 -100

17. If the mean of a data is 27 and its median is 33 then mode is 1


(a) 30 (b) 43 (c) 45 (d) 47

18. Which of the following can be the probability of an event? 1


(a) – 0.4 (b) 1.004 (c) 18/23 (d) 10/7

19. Assertion: 2 is an example of a rational number. 1


Reason : The square roots of all positive integers are irrational numbers.

a) Both assertion (A) and reason (R) are true and reason(R) is the correct explanation of assertion (A).
(b) Both assertion (A) and reason (R) are true but reason(R) is not the correct explanation of assertion (A).
(c) Assertion (A) is true but reason (R) is false.
(d) Assertion (A) is false but reason (R) is true
20. Assertion: The point (0, 4) lies on y-axis. 1
Reason : The x co-ordinate of the point (0, 4) is zero.

a) Both assertion (A) and reason (R) are true and reason(R) is the correct explanation of assertion (A).
(b) Both assertion (A) and reason (R) are true but reason(R) is not the correct explanation of assertion (A).
(c) Assertion (A) is true but reason (R) is false.
(d) Assertion (A) is false but reason (R) is true.
Page 2 of 6
Section B 2 x 5 = 10

21. The larger of two supplementary angles exceeds the smaller by 18 degrees. Find them. 2
22. 2 tan 30°/(1 + tan230°) = ? 2
Or
If tan (A + B) = √3 and tan (A – B) = 1√3; 0° < A + B ≤ 90°; A > B, find A and B.
23. From a point Q. the length of the tangent to a circle is 24 cm and the distance of Q from the 2
centre is 25 cm. The radius of the circle is

24. In the figure, if LM || CB and LN || CD, prove that AM/AB = AN/AD 2

25. A square is inscribed in a circle. Calculate the ratio of the area of the circle and the square. 2
Or
Calculate the area of a sector of angle 60°. Given, the circle has a radius of 6 cm.

Section C 3 x 6 =18
26. Find a quadratic polynomial whose sum and product of its zeroes are -1/4, 1/4 respectively. 3

27. Five years ago, Nuri was thrice as old as Sonu. Ten years later, Nuri will be twice as old as Sonu. 3
How old are Nuri and Sonu?
Or
If we add 1 to the numerator and subtract 1 from the denominator, a fraction reduces to 1. It becomes
1/2 , if we only add 1 to the denominator. What is the fraction?
28. Prove (tan θ + sec θ – 1)/(tan θ – sec θ + 1) = (1 + sin θ)/cos θ 3

29. Prove that the tangents drawn at the ends of a diameter of a circle are parallel. 3
Or
A quadrilateral ABCD is drawn to circumscribe a circle (Fig. given below).
Prove that AB + CD = AD + BC

Page 3 of 6
30. A box contains 5 red marbles, 8 white marbles and 4 green marbles. One marble is taken out of the 3
box at random. What is the probability that the marble taken out will be
(i) red?
(ii) white?
(iii) not green?

31. Prove that √7 is an irrational number 3

Section D 4 x 5 =20
32. (a) In ∆ABC, DE || BC, find the value of x. 2

(b) In the figure, if DE || OB and EF || BC, then prove that DF || OC.


3

33. A train travels 180 km at a uniform speed. If the speed had been 9 km/hr more, it would have 3
taken 1 hour less for the same journey. Find the speed of the train. 2
Or
A train covers a distance of 480 km at a uniform speed. If the speed had been 8 km/hr less
than it would have taken 3 hours more to cover same distance. Find the usual speed of the
train.

34. A cubical block of side 7 cm is surmounted by a hemisphere. What is the greatest diameter the 3
hemisphere can have? Find the surface area of the solid.
2
Or
2 cubes each of volume 64 cm3 are joined end to end. Find the surface area of the resulting cuboid.

35. If the median of a distribution given below is 28.5 then, find the value of x & y. 5

Page 4 of 6
Section E 3 x 4 = 12

36. India is competitive manufacturing location due to the low cost of manpower and strong technical
and engineering capabilities contributing to higher quality production runs. The production of TV
sets in a factory increases uniformly by a fixed number every year. It produced 16000 sets in 6th year
and 22600 in 9th year.

Based on the above information, answer the following questions:


(i). Find the production during first year. 1
(ii). Find the production during 8th year. 1
(iii). Find the production during first 3 years 2
Or
Find the difference of the production during 7th year and 4th year.
37. In a GPS, The lines that run east-west are known as lines of latitude, and the lines running north-
south are known as lines of longitude. The latitude and the longitude of a place are its coordinates
and the distance formula is used to find the distance between two places. The distance between two
parallel lines is approximately 150 km.
A family from Uttar Pradesh planned a round trip from Lucknow (L) to Puri (P) via Bhuj
(B) and Nashik (N) as shown in the given figure below.

Page 5 of 6
Based on the above information, answer the following questions:
1
(i) Find the distance between Lucknow (L) to Bhuj(B).
(ii) If Kota (K), internally divide the line segment joining Lucknow (L) to Bhuj (B) into 3 : 2 1
then find the coordinate of Kota (K).
(iii) Name the type of triangle formed by the places Lucknow (L), Nashik (N) and Puri (P)
Or 2

Find a place (point) on the longitude (y-axis) which is equidistant from the points Lucknow (L) and
Puri (P).
38. There are two temples on each bank of a river. One temple is 50 m high. A man, who is standing on
the top of 50m high temple, observed from the top that angle of depression of the top and foot of
other temple are 30° and 60° respectively. (Take 3 = 1.73)

Based on the above information, answer the following questions:

(i) . Measure of angle ∠ADF is equal to


(a) 45° (b) 60° (c) 30° (d) 90° 1

(ii) Width of the river is

(a) 28.90 m (b) 26.75 m (c) 25 m (d) 27 m 2


Or
Height of the other temple is
(a) 32.5 m (b) 35 m (c) 33.33 m (d) 40 m

(iii). Height of the other temple is

(a) reflex angle (b) straight angle (c) an obtuse angle (d) an acute angle 1

Page 6 of 6
KENDRIYA VIDYALYA SANGATHAN, JAMMU REGION
Sample Paper Set - 14

CLASS – X SUBJECT -Mathematics Basic (241)


Time : 3 Hours MM: 80
General Instructions:

1. This Question Paper has 5 Sections A, B, C, D, and E.


2. Section A has 20 Multiple Choice Questions (MCQs) carrying 1 mark each.
3. Section B has 5 Short Answer-I (SA-I) type questions carrying 2 marks each.
4. Section C has 6 Short Answer-II (SA-II) type questions carrying 3 marks each.
5. Section D has 4 Long Answer (LA) type questions carrying 5 marks each.
6. Section E has 3 Case Based integrated units of assessment (4 marks each) with sub-parts of the values of 1, 1
and 2 marks each respectively.
7. All Questions are compulsory. However, an internal choice in 2 Qs of 2 marks, 2 Qs of 3 marks and 2
Questions of 5 marks has been provided. An internal choice has been provided in the 2 marks questions of
Section E.

Section A 1 X 20 = 20

1. Two positive integers A and B can be expressed as A = xy3 and B = x4y2z; x, y being prime numbers then
HCF (A, B) is
(a) xy² (b) x4y²z (c) x4y3 (d) x4y3z

2. There are 312, 260 and 156 students in class X, XI and XII respectively. Buses are to be hired to take these
students to a picnic. Find the maximum number of students who can sit in a bus if each bus takes equal
number of students
(a) 52 (b) 56 (c) 48 (d) 63

3.A polynomial of degree n has:


(a) Exactly n zeroes(b) At least n zeroes(c) More than n zeroes(d) only one zero

4. The system of equation 2x + 3y – 7 = 0 and 6x + 5y – 11 = 0 has


(a) unique solution(b) No solution(c) Infinitely many solutions(d) None of these

5.The equation 2x² + kx + 3 = 0 has two equal roots, then the value of k is
(a) ±√6(b) ± 4(c) ±3√2(d) ±2√6

6.If the distance between the points (2, –2) and (–1,k ) is 5, one of the values of is
(a) –2 (b) 2 (c) –1 (d) 1

7.Find the point on X-axis which is equidistant from the points (-2, 5) and (2, -3).
(a) (1, 1) (b) (1, 0) (c) (-2, 0) (d) (3, 0)

Page 1 of 6
8.If ABC is right angled at C , then the value of sec(A + B) is

(a) 1 (b) 0 (c) √3/2 (d) not defined

9. What happens to value of CosA when A increases from 0º to 90º.


(a) decreases from 1 to 0. (b) increases from 0 to 1.
(c) increases from 1/2 to 1. (d) decreases from 1 to ½

10.If sin A = 8/17 then what is the value of cos A?


a) 8/17 (b) 15/17 (c) 8/15 (d) 17/8

11.In ΔABC, AC = 15 cm and DE || BC. If AB/AD=3, Find EC.


(a) 5cm (b) 10 cm (c) 2.5cm (d) 9cm

12. The length of a tangent drawn from a point to a circle is at a distance of 5 cm of circle is 4cm. The radius
of the circle is
(a) 4 cm(b) 5 cm(c) 3 cm(d) 7 cm
13.The distance between two parallel tangents of a circle of radius 4 cm is
(a) 2 cm(b) 4 cm(c) 6 cm(d) 8 cm

14.Perimeter of a sector of a circle whose central angle is 90° and radius 7 cm is.
(a) 35 cm(b) 25 cm(c) 77 cm(d) 7 cm

15. It is required to make a closed cylindrical tank of height 1 m and base diameter 140cm from a metal
sheet. How many square meters a sheet is required for the same?
(a) 6.45m² (b) 6.48m² (c) 7.48m² (d) 5.48m²

16. The mean of first n natural no is:

(a) n (b) n(n+1)/2 (c) (n+1)/2 (d) none of these

17.Relation between mean, mode and median is:


(a) Mean =3median-2 mode (b) median=3 mode-2mean
(c) mode =3median-2mean (d) mode =3median+2mode

18. When a die is thrown, the probability of getting an odd number less them 3 is:

(a) 1/6(b) 1/3(c) 1/2(d) 0

Directions: In the following questions, a statement of assertion (A) is followed by a statement of reason
(R). Mark the correct choice as:

19. Assertion(A): √5 is an irrational number.


Reason(R) : The square root of all positive integers is irrationals.
(a) Both assertion (A) and reason (R) are true and reason (R) is the correct explanation of assertion (A).
(b) Both assertion (A) and reason (R) are true but reason (R) is not the correct explanation of assertion (A).
(c) Assertion (A) is true but reasons (R) is false.
(d) Assertion (A) is false but reasons (R) is true

Page 2 of 6
20.Assertion(A): The points A(-2,1) , B(1,0) and C (4,-1) are collinear.
Reason(R) : Three points A B and C are collinear in same straight line, if AB+BC=AC

(a) Both assertion (A) and reason (R) are true and reason (R) is the correct explanation of assertion (A).
(b) Both assertion (A) and reason (R) are true but reason (R) is not the correct explanation of assertion (A).
(c) Assertion (A) is true but reasons (R) is false.
(d) Assertion (A) is false but reasons (R) is true

Section B 2 x 5 = 10
21.For what value of k, the following pair of linear equations may have unique solution?

2x+k y=7
3x+9y=13

22. If cosecA = √5, find the value of cotA - cosA?


Or

If sin A = 3/4, calculate cos A and tan A.

23.Find the value of x for which DE║AB

24. In the given figure, O is the centre of a circle, AB is a chord and AT is the tangent at A.
If ∠AOB = 100°, then calculate ∠ BAT

25.Find the area of the sector of a circle with a radius of 4 cm and of angle 30°. Also, find the area of the
corresponding major sector (Use π = 3.14)
Or

Find the angle subtended at the centre of a circle of radius 5 cm by an arc of length 5π/3 cm

Page 3 of 6
Section C 3 x 6 = 18
26.Prove that √ 7 is irrational?

27. Find the zeros of the quadratic polynomial x²+8x+16 and verify the relationship between the zeros and
the coefficients.

28.Determine graphically the vertices of the triangle formed by the lines representing the equations x+ y=5,
x-y=5 and Y- axis?
Or

Solve 2x + 3y = 11 and 2x – 4y = – 24 and hence find the value of ‘m’ for which y = mx + 3.

29.If 7Sin²A+ 3Cos²A=4 then show that tanA=1/√3.

30. In the given figure, the sides AB, BC and CA of a triangle ABC touch a circle at P, Q and R respectively.
If PA = 4 cm, BP = 3 cm and AC = 11 cm, find the length of BC (in cm)

31. Two dice are thrown simultaneously. Find the probability of getting:

a)a doubletb)sum is less than or equal to 12c)sum is equal to 13

Section D 5 x 4 = 20
32. A shopkeeper buys some books for 80. If he had bought 4 more books for the same amount, each book
would have cost Re.1 less. Find the number of books he bought.
Or

The numerator of a fraction is 3 less than its denominator. If 2 is added to both the numerator and the
denominator, then the sum of the new fraction and original fraction is a 2920. Find the original fraction.

33.In the figure, altitudes AD and CE of ΔABC intersect each other at the point P. Show

that:
(i) ΔAEP ~ ΔCDP (ii) ΔABD ~ ΔCBE
(iii) ΔAEP ~ ΔADB (iv) ΔPDC ~ ΔBEC
(2+1+1+1)
Page 4 of 6
34.Arul has to make arrangements for the accommodation of 150 persons for his family function. For this
purpose, he plans to build a tent which is in the shape of cylinder surmounted by a cone. Each person
occupies 4 sq. m of the space on ground and 40 cu. meter of air to breathe. What should be the height of
the conical part of the tent if the height of cylindrical part is 8 m?
Or
A cylinder of volume 150 cu.cm is placed with a cone, whose height is 4cm. If the height of cone and
cylinder is equal, then find the total volume of shape formed by the combination of cylinder and cone.

35. In a class test, marks obtained by 120 students are given in the following frequency distribution. If it is
given that mean is 59, find the missing frequencies x and y.
Marks Number of students

0-10 1

10-20 3

20-30 7

30-40 10

40-50 15

50-60 x

60-70 9

70-80 27

80-90 18

90-100 y

Section E 4 x 3 = 12
36. In a class the teacher asks every student to write an example of A.P. Two friends Geeta and Madhuri
writes their progressions as -5, -2, 1,4, ... and 187, 184, 181, .... respectively. Now, the teacher asks various
students of the class the following questions on these two progressions. Help students to find the answers of
the questions.

(i) Find the 34th term of the progression written by Madhuri. (1)
(ii) Find the sum of common difference of the two progressions. (1)
(iii)Find the sum of first 10 terms of the progression written by Geeta.
Or (2)
Which term of the two progressions will have the same value?

Page 5 of 6
37. A farmer Naresh has the field AGDCFE as shown in the map. He decided to grow three crops in his field.
Naresh divided his field into three parts for each crop. There is shortage of onions in the country and he
decided to grow onions on the field 1. In the field 2 he decided to grow wheat and in remaining field 3 he
decided to grow tomatoes.

Map showing field owned by a Naresh FARMER

(i) What is the length of the common boundary shared by onion and wheat? (1)
(ii) In how much area wheat is grown? (1)
(iii) In how much area onion is grown?
Or (2)
In what ratio does B divide line FC?

38.

Tower cranes are a common fixture at any major construction site. They’re pretty hard to miss – they often
rise hundreds of feet into the air, and can reach out just as far. The construction crew uses the tower crane to
lift steel, concrete, large tools like acetylene torches and generators, and a wide variety of other building
materials. A crane stands on a level ground. It is represented by a tower AB, of height 24 m and a jib BR.
The jib is of length 16 m and can rotate in a vertical plane about B. A vertical cable, RS, carries a load S. The
diagram shows current position of the jib, cable and load.

(i)What is the distance BS? (1)


(ii)What is the angle that the jib, BR, makes with the horizontal? (1)
(iii)What is the measure of the angle BRS?
Or (2)
What is the distance AS?

Page 6 of 6
KENDRIYA VIDYALYA SANGATHAN, JAMMU REGION
Sample Paper Set - 15

CLASS – X SUBJECT -Mathematics Basic (241)


Time : 3 Hours MM: 80
General Instructions:

1. This Question Paper has 5 Sections A, B, C, D, and E.


2. Section A has 20 Multiple Choice Questions (MCQs) carrying 1 mark each.
3. Section B has 5 Short Answer-I (SA-I) type questions carrying 2 marks each.
4. Section C has 6 Short Answer-II (SA-II) type questions carrying 3 marks each.
5. Section D has 4 Long Answer (LA) type questions carrying 5 marks each.
6. Section E has 3 Case Based integrated units of assessment (4 marks each) with sub-parts of the values of 1, 1
and 2 marks each respectively.
7. All Questions are compulsory. However, an internal choice in 2 Qs of 2 marks, 2 Qs of 3 marks and 2
Questions of 5 marks has been provided. An internal choice has been provided in the 2 marks questions of
Section E.

Section A 1 x 20 = 20
1. HCF of 168 and 126 is 1
(a) 21 (b) 42 (c) 14 (d) 18
2. Express 98 as a product of its primes 1
(a) 2² × 7(b) 2² × 7²(c) 2 × 7²(d) 23 × 7

3. The sum of the zeroes of the polynomial 2x²-8x +6 is 1


(a) – 3 (b) 3 (c) - 4 (d) 4
4. The pair of lines represented by the equations 2x+y+3 = 0 and 4x+ky+6 = 0 will be parallel if 1
value of k is ______.
a)1 b)-2 c)2 d)3

5. If the quadratic equation x²-2x + k = 0 has equal roots, then value of k 1


a)1 b)2 c)-1 d)-3

6. The distance between the point P(1, 4) and Q(4, 0) is 1


(a) 4(b) 5(c) 6(d) 3√3
7. The distance of the point P(2, 3) from the x-axis is 1
(a) 2(b) 3(c) 1(d) 5

8. The probability of event equal to zero is called; 1


(a) Unsure event (b) Sure Event
(c) Impossible event (d) Independent event
9. The value of tan 60°/cot 30° is equal to: 1
(a) 0 (b) 1 (c) 2 (d) 3
10. 1 – cos2A is equal to: 1
(a) sin²A (b) tan²A (c)cos²A(d) sec²A
11. The value of (sin 45° + cos 45°) is 1
(a) 1/√2 (b) √2 (c) √3/2 (d) 1

Page 1 of 6
12. All circles are ……………. . 1
a)Congruent b) similar c)polygon d) none of these

13. A circle has a number of tangents equal to 1


(a) 0(b) 1(c) 2(d) Infinite
14. The area of a sector of a circle with radius 6 cm if the angle of the sector is 60°. 1
(a) 142/7(b) 152/7(c) 132/7(d) 122/7

15. Area of the circle with radius 5cm is equal to:


(a) 60 sq.cm(b) 75.5 sq.cm(c) 78.5 sq.cm (d) 10.5 sq.cm

16. If r is the radius of the sphere, then the surface area of the sphere is given by; 1
(a) 4 π r (b) 2 π r (c) π r (d) 4/3 π r
2 2 2 2

17. The relationship between mean, median and mode for a moderately skewed distribution is 1
(a) mode = median – 2 mean
(b) mode = 3 median – 2 mean
(c) mode = 2 median – 3 mean
(d) mode = median – mean

18. Mode is the 1


(a) middle most frequent value
(b) least frequent value
(c) maximum frequent value(d) none of these

19. Assertion :The distance point P(2,3) from the x-axis is 3. 1

Reason: The distance from x-axis is equal to its ordinate.

a) Both assertion (A) and reason (R) are true and reason(R) is the correct explanation of assertion (A).
(b) Both assertion (A) and reason (R) are true but reason(R) is not the correct explanation of assertion (A).
(c) Assertion (A) is true but reason (R) is false.
(d) Assertion (A) is false but reason (R) is true
20. 1
Assertion: 3 x 5 x 7 + 7 is a composite number.
Reason: A composite number has factors one, itself and any other natural number.
a) Both assertion (A) and reason (R) are true and reason(R) is the correct explanation of assertion (A).
(b) Both assertion (A) and reason (R) are true but reason(R) is not the correct explanation of assertion (A).
(c) Assertion (A) is true but reason (R) is false.
(d) Assertion (A) is false but reason (R) is true

Section B 2 x 5 = 10

21. Solve the following pair of linear equations ; 2


x + y = 14
x–y=4

Page 2 of 6
22. Evaluate 2 tan2 45° + cos2 30° – sin2 60°. 2
Or
If sin θ + cos θ = √3, then prove that tan θ + cot θ = 1.

23. In ∆DEW, AB || EW. If AD = 4 cm, DE = 12 cm and DW = 24 cm, then find the value of DB. 2

24. A quadrilateral ABCD is drawn to circumscribe a circle (see figure). 2


Prove that AB + CD = AD + BC.

25. A square is inscribed in a circle. Calculate the ratio of the area of the circle and the square. 2
Or
Calculate the area of a sector of angle 60°. Given, the circle has a radius of 6 cm.
Section C 3 x 6 =18
26. Prove that , √5 is an irrational number. 3

27. Find the zeroes of p(x) = 2x2 – x – 6 and verify the relationship of zeroes with these coefficients. 3

28. For what value of k, the pair of equations 4x – 3y = 9, 2x + ky = 11 has no solution? 3


Or
Solve 2x + 3y = 11 and 2x – 4y = – 24 and hence find the value of ‘m’ for which y = mx + 3.

29. Prove(sec A + tan A) (1 – sin A)= cos A 3


30. Prove that the lengths of the tangents drawn from an external point to a circle are equal. 3
Or
The length of a tangent from a point A at distance 5 cm from the centre of the circle is 4 cm. Find
the radius of the circle.
31. A die is thrown once. Find the probability of getting 3
(i) a prime number
(ii) a number lying between 2 and 6
(ill) an odd number.
Section D 4 x 5 =20
32. The sum of the areas of two squares is 468 m2. If the difference of their perimeters is 24 m, find 5
the sides of the two squares.
Or
If Zeba was younger by 5 years than what she really is, then the square of her age (in years)
would have been 11 more than five times her actual age. What is her age now?

33. State and prove basic proportionality theorem. 3


2
Also In the given figure, XY || QR, PQXQ=73 and PR = 6.3 cm, find YR.

Page 3 of 6
34. A medicine capsule is in the shape of a cylinder with two hemispheres stuck to each of its ends. 5
The length of the entire capsule is 14 mm and the diameter of the capsule is 5 mm. Find its
surface area.

Or
A tent is in the shape of a cylinder surmounted by a conical top. If the height and diameter of the
cylindrical part are 2.1 m and 4 m respectively, and the slant height of the top is 2.8 m, find the
area of the canvas used for making the tent. Also, find the cost of the canvas of the tent at the rate
of Rs 500 per m2. (Note that the base of the tent will not be covered with canvas.)
35. 100 surnames were randomly picked up from a local telephone directory and the frequency 5
distribution of the number of letters in the English alphabet in the surnames was obtained as
follows:

Determine the median number of letters in the surnames. Find the mean number of letters in the
surnames. Also, find the modal size of the surnames.

Section E 3 x 4 = 12

36. In order to conduct Sports Day activities in your School, lines have been drawn with chalk
powder at a distance of 1 m each, in a rectangular shaped ground ABCD, 100 flower pots have
been placed at a distance of 1 m from each other along AD, as shown in given figure below.
Niharika runs 1/4 th the distance AD on the 2nd line and posts a green flag. Preet runs 1/5th
distance AD on the eighth line and posts a red flag.

1. Find the position of green flag 1

a) (2,25)b) (2,0.25)c) (25,2)d) (0, -25)

2.Find the position of red flag 1

Page 4 of 6
a) (8,0) b) (20,8) c) (8,20) d) (8,0.2)

3. What is the distance between both the flags?


a) √41 b) √11 c) √61 d) √51 2
Or
If Rashmi has to post a blue flag exactly halfway between the line segment joining the two flags,
where should she post her flag?

37. Your friend Veer wants to participate in a 200m race. He can currently run that distance in 51
seconds and with each day of practice it takes him 2 seconds less. He wants to do in 31 seconds.

1. Which of the following terms are in AP for the given situation 1

a) 51,53,55….b) 51, 49, 47….c) -51, -53, -55….d) 51, 55, 59…

2.What is the minimum number of days he needs to practice till his goal is achieved
a) 10 b) 12 c) 11 d) 9 2
Or

Find the value of x , if 2x , x + 10 and 3x + 2 are three consecutive terms of an AP.


a) 10 b) 6 c) 7 d) -10
1
3. If nth term of an AP is given by an = 2n + 3 then common difference of an AP is
a) 2b) 3c) 5d) 1

38. A group of students of class X visited India gate on an education trip the teacher and students had
interested in history as well. the narrate the India gate. Official name Delhi Memorial originally
called All- India War Memorial, monumental sand stone arch in new Delhi dedicated to the
troops of British India who died in wars fought between 1914 and 1919. The teacher also said
that india gate, which is located at the eastern end of the Rajpath (formerly called the Kingsway)
is about 138 feet (42 metres) in height.

(i) The ratio of the length of a Rod and its shadow is 1:1. The angle of elevation of the sun
1
Page 5 of 6
is?
a) 300b) 450c) 600d) 900

(ii) The angle formed by the line of sight with the horizontal when the object viewed is
1
below the horizontal level is
a) corresponding angleb) angle of elevation
c) angle of depressiond) complete angle
(iii) if the altitude of the sun is at 60º. then the height of the vertical tower that will cast a shadow
2
of length 20 m is ?

Or
What is the angle of elevation if they are standing at a distance of 42m away from the
monument?

(a) 30 º (b) 45 º (c) 60 º (d) 0 º

Page 6 of 6

You might also like